You are on page 1of 258

HEMATOLOGY

BOARD REVIEW
Alfadel Alshaibani
Consultant Hematologist at King Faisal Specialist Hospital-Riyadh
INTRODUCTION
 Questions may look hard, don’t be discouraged, the purpose of this lecture is to prepare you
for the board NOT to test you.
 The lecture is exam-oriented. Some of the generalization mentioned may need further details
in real life, but for exam purposes, I hope this is all what you need.
1) A 40-year-old female presents to an emergency department (ED) for evaluation of recurrent abdominal pain,
nausea, vomiting, diarrhea, and progressive bilateral lower extremity weakness. She has been hospitalized
with similar complaints at least ten times in the last one and half years without a specific diagnosis. During
her last admission, she was treated for acute gastroenteritis with empiric antibiotics. Her evaluation was
negative, including an abdominal CT and upper and lower endoscopy.
Today, she had a witnessed seizure in the ED that lasted for 2 minutes. Her blood chemistry is notable for
severe hyponatremia with a sodium of 120 mEq/L and mildly elevated serum transaminases. She is tachycardic
with a heart rate of 125 beats/min, and her blood pressure is 165/95 mmHg. The physical exam is notable for
generalized, mild abdominal tenderness without any guarding, rigidity, or rebound tenderness. The neurologic
exam is significant for disorientation and 2/5 strength and decreased sensation in her lower extremities. Her CT
head is negative for acute injury. An EEG has been ordered, and neurology has been consulted. She is admitted
to the intensive care unit for further management. Besides correcting electrolytes abnormalities and
management of acute problems, what else should be considered?
a) MRI brain to evaluate for the cause of her seizure
b) Urine test for porphobilinogen (PBG)
c) Consult psychiatry for evaluation of a psychiatric cause
d) Repeat EGD (esophagogastroduodenoscopy) and colonoscopy
e) Peripheral blood smear and vitamin B12 level.
a) MRI brain to evaluate for the cause of her seizure
b) Urine test for porphobilinogen (PBG)
c) Consult psychiatry for evaluation of a psychiatric cause
d) Repeat EGD (esophagogastroduodenoscopy) and colonoscopy
e) Peripheral blood smear and vitamin B12 level.
 Whenever you hear the classic triad of abdominal pain, CNS abnormalities and peripheral
neuropathy … THINK acute intermittent porphyria.

 Acute intermittent porphyria is caused by porphobilinogen deaminase). 


 Autosomal dominant disease with (low penetrance). 

 Always look for exacerbating factors :medications (almost always antiseizure), menstrual cycle,
alcohol, fasting and stress. 

 Diagnosis: urine porphobilinogen. 


2) A 17-year-old male came to the emergency department with abdominal pain, seizures, and
episodes of vomiting. Morphine and diazepam were given. Urine showed increased porphobilinogen.
No more seizures episodes were reported. Abdominal pain is decreasing with the therapy. Which is
the next step in the treatment of this condition?
 
a) Stop morphine and reassure
b) Give intravenous heme
c) Change diazepam to valproic acid
d) Prepare for a liver transplant
2) A 17-year-old male came to the emergency department with abdominal pain, seizures, and
episodes of vomiting. Morphine and diazepam were given. Urine showed increased porphobilinogen.
No more seizures episodes were reported. Abdominal pain is decreasing with the therapy. Which is
the next step in the treatment of this condition?
 
a) Stop morphine and reassure
b) Give intravenous heme
c) Change diazepam to valproic acid
d) Prepare for a liver transplant
Treatment of acute porphyria.: 

Severe: Hemin (can be given to pregnant patients)

Mild: IV hemin Vs carbohydrate loading ( IV dextrose)

Prophylaxis: 
More than 4 attackes per year : Givosiran.  GnRH agonists for menstrual induced attacks.

Definitive treatment: liver transplant


3) A 65-year-old male with a history of acute intermittent porphyria (AIP) presented to his
healthcare provider for a routine checkup. His last attack was 5 years ago. His physical exam
reveals no abnormalities. Lab values are within the normal range. Due to his condition, he can
have an increased risk of a type of cancer. Which surveillance methods should be adopted for
this patient?
 
a) Liver ultrasound and alpha-fetoprotein
b) Prostatic specific antigen (PSA) and digital rectal exam
c) Low dose CT thorax
d) Colonoscopy
e) Bone marrow biopsy.
3) A 65-year-old male with a history of acute intermittent porphyria (AIP) presented to his
healthcare provider for a routine checkup. His last attack was 5 years ago. His physical exam
reveals no abnormalities. Lab values are within the normal range. Due to his condition, he can
have an increased risk of a type of cancer. Which surveillance methods should be adopted for
this patient?
 
a) Liver ultrasound and alpha-fetoprotein
b) Prostatic specific antigen (PSA) and digital rectal exam
c) Low dose CT thorax
d) Colonoscopy
e) Bone marrow biopsy.
It is demonstrated that patients with AIP have an increased risk for hepatic carcinoma after the
age of 50, and they should have surveillance tests performed regularly.

Surveillance tests include performing a liver ultrasound and alpha-fetoprotein every 6-12
months. One of the hypotheses is that chronic elevation of delta-aminolevulinic acid can lead to
free radicals with subsequent effects on DNA that can cause carcinogenesis.
 
Prostate, lung, and colon cancer are not increased in acute intermittent porphyria in relation to
the general population.
4) A 23-year-old woman presents with progressive sleeplessness, memory loss, and depression.
History reveals that she is a strict vegetarian who has been on a corn diet for more than 15 years.
She manifests anemia and a facial dermatitis. Which of the following is her likely diagnosis?
 
a) Zinc deficiency
b) Niacin deficiency
c) Thiamine deficiency
d) Vitamin C deficiency
e) Copper deficiency.
4) A 23-year-old woman presents with progressive sleeplessness, memory loss, and depression.
History reveals that she is a strict vegetarian who has been on a corn diet for more than 15 years.
She manifests anemia and a facial dermatitis. Which of the following is her likely diagnosis?
 
a) Zinc deficiency
b) Niacin deficiency
c) Thiamine deficiency
d) Vitamin C deficiency
e) Copper deficiency.
Niacin deficiency (B3): Pellagra which is characterized by the 3 D’s. Dementia, dermatitis,
diarrhea.
Pellagra is due to a deficiency of nicotinic acid and its precursor tryptophan. Diets are rich in
corn and deficient in animal fats. This disease typically affects the skin, digestive tract, CNS,
and hematopoietic system. Corn grains are particularly low in niacin and tryptophan
5) A 52-year-old male presents to the pain clinic for a chronic back pain follow up appointment.
The pain is described as aching, and the patient denies any radiating pain. The patient tells you
that he believes his pain is due to poor posturing during his work as a dentist, bending over
chairs 8 hours a day for 20 years. He reports he has been on a vegan diet for about a year now.
On exam, he appears pale, and his blood pressure is 100/68 mmHg, heart rate 102 beats/min,
and respiratory rate 18. A CBC is significant for a hemoglobin of 10.0 grams/dL, and mean
corpuscular volume (MCV) 108. Which agent that this patient is regularly exposed to could
have influenced these lab findings?
a) Bupivacaine
b) Nitrous oxide gas
c) Lidocaine
d) Sevoflurane gas
a) Bupivacaine
b) Nitrous oxide gas
c) Lidocaine
d) Sevoflurane gas
Chronic exposure to nitrous oxide gas causes decreased activity of the enzyme methionine synthase,
which is involved in vitamin B12 dependent metabolic pathways. Patients with chronic exposure,
classically those who work in dental offices, can present with relative vitamin B12 deficiency and
megaloblastic anemia if their diets do not correct the deficiency.
In this case, the patient reports he has been a vegan for more than a year. A vegan diet can be a cause
of B12 deficiency. This patient's diet would not correct the deficiency induced by the chronic gas
exposure.  
6) A 32-year-old female with a history of HIV infection is admitted with intermittent fever, dyspnea,
and cough. She undergoes a bronchoalveolar lavage during which she receives 10 mL of 2%
lidocaine solution instilled into the trachea as well as 2 sprays of 2% lidocaine solution.
Benzodiazepines are given for moderate sedation. Shortly after the procedure the patient develops
respiratory distress and was visibly cyanotic. Her vitals immediately after the procedure are the
following: blood pressure is 180/72 mmHg, the pulse is 140/min, and oxygen saturation is 90% with
satisfactory waveform. Which of the following diagnostic tests would be most helpful in identifying
the underlying problem?
a) CT Angiography.
b) Co-oximetry.
c) PaO2 and SaO2 on ABG
d) Chest X ray.
e) Peripheral blood smear
a) CT Angiography.
b) Co-oximetry.
c) PaO2 and SaO2 on ABG
d) Chest X ray.
e) Peripheral blood smear
 Methemoglobinemia is a rare complication associated with lidocaine use (other medications
include Dapsone, Nitrous oxide, Rasburicase).
 Early symptoms consist of respiratory distress and cyanosis despite having normal arterial pO2 and
normal saturation.
 Another exam clue is chocolate brown blood.

 In normal hemoglobin, iron is in the ferrous (Fe2+) state, which allows binding of oxygen gas (O2).

 Methemoglobin is an altered state of hemoglobin in which the heme iron is oxidized from the
ferrous (Fe2+) to the ferric (Fe3+) state.
 The ferric hemes of methemoglobin do not bind O2 , This result in functional anemia.
 Co-oximeter is a device that is able to determine SaO2 more accurately especially in the
presence of non-oxygen-carrying hemoglobin, carboxyhemoglobin, and methemoglobin.
Pulse oximetry and arterial blood gas are unreliable as they would be unable to differentiate
between oxyhemoglobin/deoxyhemoglobin and methemoglobin.
7) A 16-year-old male has noticed that he has been voiding tea-colored urine in the morning for
the past few weeks. He also complains of fatigue, mild dyspnea, and abdominal pain. Physical
exam reveals an ill-looking patient with painful hepatomegaly. His lactate dehydrogenase level
is increased and there is a moderate normochromic anemia with a description of polychromasia
on the blood smear. CT abdomen showed portal vein thrombosis. Flow cytometry reveals an
absence of glycosylphosphatidylinositol-anchored proteins in the red cells. Which of the
following is the most likely diagnosis?
a) Fanconi anemia
b) Paroxysmal nocturnal hemoglobinuria
c) Acute myeloid leukemia
d) Hereditary spherocytosis
e) Antiphospholipid syndrome.
a) Fanconi anemia
b) Paroxysmal nocturnal hemoglobinuria
c) Acute myeloid leukemia
d) Hereditary spherocytosis
e) Antiphospholipid syndrome.
Hemolysis, thrombosis and dark urine: think PNH. 

Thrombosis typically occurs in unusual space. (abdominal or cerebral vein)

Associated with aplastic anemia ( we typically screen for PNH in patients with newly diagnosed
aplastic anemia. 
Complement system activation
causes formation of membrane
attack complex that causes lysis
of cells.

Glycosylphosphatidylinositol
anchor links complement
inhibitor CD55 and CD59,
which  inhibits complement
activation. 
 lack of this causes hemolysis. 

For exam purposes:


PIGA mutation =
Glycosylphophatidylinositol
(GPI) = CD55 &CD59
8) A Patient being evaluated for anemia has the peripheral blood smear shown. Which of the
following is the most likely cause of the anemia?
A) Acute lymphocytic leukemia.
B) Autoantibodies to ADAMTS-13.
C) Cobalamin Deficiency.
D) EBV infection.
E) Iron deficiency
A Patient being evaluated for anemia has the peripheral blood smear shown. Which of the
following is the most likely cause of the anemia?
A) Acute lymphocytic leukemia.
B) Autoantibodies to ADAMTS-13.
C) Cobalamin Deficiency.
D) EBV infection.
E) Iron deficiency
C) COBALAMIN DEFICIENCY

 Findings in Megaloblastic anemia: Macrocytosis and hypersegmented neutrophil.

 Hypersegmented neutrophil : 6 or more lobes per neutrophil.

 Causes of megaloblastic anemia: vitamin B12 deficiency and folate.


9) A 31-year-old male with a past medical history of sickle cell disease presents to the clinic to
establish care. He has a history of recurrent invasive infections with encapsulated organisms.
Doppler sonography of the abdomen is significant for a small spleen with the absence of
parenchymal vascularization. Which of the following findings are most likely to be seen on the
peripheral blood smear?
a) Schistocytes
b) Spherocytes
c) Howell-Jolly bodies
d) Auer rods
9) A 31-year-old male with a past medical history of sickle cell disease presents to the clinic to
establish care. He has a history of recurrent invasive infections with encapsulated organisms.
Doppler sonography of the abdomen is significant for a small spleen with the absence of
parenchymal vascularization. Which of the following findings are most likely to be seen on the
peripheral blood smear?
a) Schistocytes
b) Spherocytes
c) Howell-Jolly bodies
d) Auer rods
Functional asplenism is characterized by the loss of phagocytic activity while retaining splenic
tissue.
Patients with sickle cell disease typically have functional asplenism (this is due to repeated splenic
infarction) leading to hyposplenism. This is sometimes referred to as autosplenectomy.

The presence of Howell-Jolly bodies on a peripheral blood smear is indicative of lack of spleen
function. . Howell-Jolly bodies are nuclear remnants of old RBCs that are typically removed by the
spleen.  
HOWELL JOLLY BODIES=
ABSENCE OF SPLEEN
10) A 34-year-old female with a medical history of multiple sclerosis (MS) comes into the ED
for a headache of 1 week. She has tried tylenol, but it has not helped. Her physical exam is
normal, and no focal neurological deficits were noted. She was admitted to the neurology ward.
Her initial laboratory workup is significant for a hemoglobin level of 9.7 mg/dL, and platelet
count of 112000/µL, otherwise all other lab work was normal. Given her MS history, a brain
MRI was requested. Brain MRI reveals thrombosis of the right transverse and inferior
longitudinal sinus, without new parenchymal damage or signs of infection. In the further
workup of atypical cerebral vein thrombosis, the deficiency in protein CD 52, CD 55, and CD
59has cells was revealed. The low molecular weight heparin was started after a week; the
patient's headache is resolved. What is the preferred medical therapy that will prevent the
recurrence of thrombosis in this patient?
a) Prednisone
b) Plasmapharesis.
c) Eculizumab
d) Apixaban
e) Exchange transfusion.
a) Prednisone
b) Plasmapharesis.
c) Eculizumab
d) Apixaban
e) Exchange transfusion.
Eculizumab is a monoclonal antibody that prevents the cleavage of C5 into C5a and C5b. C5b binds
other complement factors from C6-C9 and forms membrane attack complex, which causes cell lysis,

Hence, Eculizumab reduces intravascular hemolysis and the patients' transfusion requirements by
blocking hemolysis through the complement pathway.

Because eculizumab prevents membrane attack complex from forming, it had an adverse effect of
increased risk of infection by encapsulated bacteria, especially Neisseria. Hence all patients before
they are started on eculizumab must be vaccinated against Neisseria.
Thrombosis is the number one cause of mortality in patients with PNH so this patient should be
started on eculizumab. The best treatment that prevents recurrence of thrombosis in patients with
PNH is actually eculizumab, not new oral anticoagulation medications. This is because in PNH
pathophysiology of thrombosis and hemolysis is complement-mediated. Even though apixaban will
help in preventing thrombosis, eculizumab is shown to be superior in preventing thrombosis in
patients with PNH.
11) A 16-year-old female was admitted to the hospital five days ago complaining of fever with chills
and rigors, lumbar pain, and dysuria. Her blood pressure was 95/75 mmHg, pulse 112/min and
temperature 39.5 C. A urinalysis was performed which revealed positive leucocytes and nitrites. A
complete blood count showed hemoglobin 12.5 g/dL, MCV 82 fl, WBC 15,300/microL, and
platelets 190,000/microL. She was diagnosed as a case of pyelonephritis and was started on IV fluids
and IV ceftriaxone. She started showing signs of improvement and was afebrile by the third day of
treatment. During the past 24 hours, however, she developed severe tachycardia, pallor, and
dizziness. A complete blood count was repeated which shows hemoglobin 7.3 g/dL, MCV 97 fl,
WBC 9,500/microL and platelets 260,000/microL. A blood smear was performed which shows the
presence of spherocytes. What is the best initial therapy for the condition of this patient?
a) IV immunoglobulin (IVIG)
b) Azathioprine
c) Rituximab
d) Prednisone
a) IV immunoglobulin (IVIG)
b) Azathioprine
c) Rituximab
d) Prednisone
SPHEROCYTES

Another clue for spherocytosis is high


MCHC (typically elevated in
spherocytosis).
Sometime they will mention RBC without
central pallor = spherocytes
The diagnosis here is autoimmune hemolytic anemia (IgG-mediated). IgG antibody binds red
blood cells in the relatively warm temperature of the central body. The membrane of antibody-
coated red cells is consumed by the splenic macrophages resulting in spherocytes and eventually
hemolysis.
Cephalosporins like ceftriaxone can cause autoimmune hemolytic anemia. The drug normally
attaches to the membrane of red blood cells with subsequent binding of antibody (IgG) to the
drug-membrane complex. The diagnosis is made by the Coombs test which detects IgG antibody
on the surface of red blood cells.
The best initial therapy is prednisone. The causative drug should also be stopped right away. If
the episodes are recurrent, splenectomy may be considered. Rituximab is used when
splenectomy fails to control the hemolysis. Azathioprine is an alternative to diminish the need
for steroids in general.
 Causes of secondary warm AIHA: Infections (HIV, EBV, HCV) SLE, RA, CLL and
lymphoma.
 Treatment for warm autoimmune hemolytic anemia:
 Steroid +/- Rituximab

 > Remember IVIG has limited efficacy when it comes to warm autoimmune hemolytic
anemia.
12) A 42-year-old woman complains of fatigue that has been present for the past 6 months. Her
menstrual periods are regular, and there is no excessive blood loss. She denies palpitations, syncope,
weight loss, or fever. On examination, BP: 123/81 mmHg, HR: 72 bpm, RR:16/min, and O2: 98% on
room air. Her conjunctiva is pale and abdominal examination reveals splenomegaly.
Lab tests: HB: 9.7 g/dL (normal: 12-15 g/dL), reticulocytes: 4% (normal 0.5-2.5%), TIBC: 200
(normal: 240-500), MCHC 39 g/dL (normal: 33.4-35.5 g/dL), MCV: 82 fL/red cell (normal 80-
96fL/red cell), MCH: 30 pg/cell (normal: 27-33 pg/cell). A peripheral blood smear shows RBC
's with no central pallor. Which of the following is the most appropriate treatment for this patient?
a) Eculizumab
b) prednisone
c) Splenectomy
d) Iron supplementation
e) Hydroxyurea
a) Eculizumab
b) prednisone
c) Splenectomy
d) Iron supplementation
e) Hydroxyurea
 This patient has anemia symptoms, high MCHC (clue to the presence of spherocytes), Peripheral blood
smear showing RBC with no central pallor (another ward for spherocytes), long history (makes hereditary
cause more likely): The diagnosis is : Hereditary spherocytosis.

In the stem they either write coombs test negative or give you a long history of disease (which makes
autoimmune process less likely).

Hereditary spherocytosis is caused by autosomal dominant mutations in the genes that encode for RBC
cytoskeleton proteins such as ankyrin, band 3, protein 4.2, and spectrin.

Defects in these proteins impair the elasticity of the RBC membrane. Rigid RBC have spherocytic shape
instead of the usual RBC biconcave shape. The elasticity of the RBC membrane is essential to its survival
in the circulation, especially when it passes through the microcirculation of the spleen. A normal elastic
RBC can pass through the areas of microcirculation and squeeze itself through without bursting
(hemolyzing) and then restores its shape after passing through the microcirculation region. Defects in the
cytoskeleton proteins, as seen in hereditary spherocytosis, leads to a rigid RBC membrane which is prone
to hemolysis as it is passing through the regions of microcirculation, especially in the spleen.
 This is why patients with hereditary spherocytosis present with hemolytic anemia. Patients with
hereditary spherocytosis will have manifestations of anemia, which includes fatigue, pallor, and
delayed capillary refill time. Lab tests are consistent with hemolytic anemia: low Hb, elevated
reticulocyte count, elevated indirect bilirubin, low TIBC, decreased haptoglobin, and increased
LDH. The presence of spherocytes on a peripheral blood smear manifested as RBC's with no
central pallor, also guides towards the diagnosis. Coombs test must be done to exclude
autoimmune causes of hemolysis, as the test result should be negative in patients with hereditary
spherocytosis. The osmotic fragility test is a test where the patients RBC's are put in a hypotonic
solution and monitored for hemolysis. The presence of hemolysis confirms the diagnosis of
hereditary spherocytosis. The goal of treatment is to minimize the complications of chronic
hemolysis and anemia. Folic acid supplementation can be useful, as patients with chronic
hemolysis have a deficiency in this vitamin. Blood transfusions may be given as temporary relief
of moderate-severe hemolysis, but should not be given for a long period of time to avoid iron
overload. The best treatment is splenectomy, as it removes the reason for hemolysis. The RBC's
hemolyze as they pass through the splenic microcirculation, and removing the spleen will greatly
decrease the level of hemolysis. An increase in Hb and a decrease in reticulocyte count and indirect
bilirubin can be seen several days after splenectomy.
13) A 65-year-old man presents complaining of weakness, mild lower abdominal pain, and
darkening urine. He was treated at an urgent care center two days ago with trimethoprim-
sulfamethoxazole for suspected prostatitis. His past medical history is unremarkable. He is
afebrile and vitally stable. His blood work reveals a WBC of 10,000, hemoglobin of 9 g/dL,
MCV of 90 fl, and platelets of 200,000. His reticulocyte count is 12%. His LDH and indirect
bilirubin are elevated. His haptoglobin is low at 5 mg/dl; however, his urine dipstick is positive
for hemoglobin. His creatinine is normal. Further investigations reveal a negative direct
antiglobulin test and a negative G6PD screen. His peripheral smear reveals cells that have a
chunk of their membranes "bitten" off. What is the most likely diagnosis?
a) Thrombotic thrombocytopenic purpura (TTP)
b) Glucose-6-phosphate dehydrogenase deficiency (G6PD deficiency)
c) Sickle cell anemia
d) Autoimmune hemolytic anemia (AIHA)
a) Thrombotic thrombocytopenic purpura (TTP)
b) Glucose-6-phosphate dehydrogenase deficiency (G6PD deficiency)
c) Sickle cell anemia
d) Autoimmune hemolytic anemia (AIHA)
 This patient has hemolytic anemia (corrected reticulocyte count is elevated) secondary to treatment
with trimethoprim-sulfamethoxazole, a known precipitant of hemolysis in those with G6PD
deficiency. In G6PD deficiency, oxidative stress causes the hemoglobin to denature and precipitate
within the cells, and form Heinz bodies. These abnormal precipitants are recognized by
macrophages and "pitted" out, giving the appearance of a bite cell.
 It is important to note that during acute hemolysis G6PD is NEGATIVE because reticulocyte count
is elevated because younger red cells are not deficient in this enzyme compared to older red cells.
 If the test were to be repeated after the acute crisis and resolution of the reticulocytosis, it should
be abnormal.
 TTP causes the formation of "schistocytes" which is a red blood cell (RBC) with increased cell
fragility, that is fragmented as it traverses through damaged blood vessels. AIHA results in the
formation of spherocytes. These cells are usually sphere-shaped, rather than the regular bi-concave
discs that represent an RBC.
BITTEN CELLS (RIGHT) HEINZ
BODIES (LEFT)
14) A 51-year-old postmenopausal woman presents for follow-up of a finding of persistent
microcytosis. The patient is asymptomatic and denies any bleeding, blood transfusion, surgery,
or previous diagnoses of anemia. Her complete blood count shows a hemoglobin of 12.3 g/dL,
mean corpuscular volume (MCV) 72.1 fl, mean corpuscular hemoglobin (MCH) 22.4 pg and a
red cell distribution width (RDW) of 14%. Her iron studies are essentially normal (transferrin
saturation 29%, ferritin 84 ng/mL). Hemoglobin electrophoresis shows a pattern of HbA1
97.1%, HbA2 2.6%, HbF 0.3%. The blood smear review describes moderate hypochromia and
poikilocytosis with regular codocytes. What is the most likely diagnosis?
a) Thalassemia intermedia
b) Congenital microcytosis
c) Alpha-thalassemia silent carrier
d) Thalassemia minor
a) Thalassemia intermedia
b) Congenital microcytosis
c) Alpha-thalassemia silent carrier
d) Thalassemia minor
Microcytic anemia + normal hemoglobin electrophoresis + normal iron level = Alpha
thalassemia trait.
 The silent carrier state for alpha thalassemia causes only microcytosis and hypochromia
without anemia. Codocytes (target cells) are a common finding in all the thalassemic states.
 The condition is clinically silent and does not require treatment. Humans have two alpha-
globin genes located on each chromosome 16, resulting in 4 a-gene loci (aa/aa). Adult
hemoglobin is a mixture of HbA1, HbA2, and HbF. All of these hemoglobins are composed of
2 alpha-globin chains and 2 non-alpha-globin chains (delta in HbA2 and gamma in HbF). The
alpha thalassemia carrier state is the result of deletion or inactivation of only one alpha-globin
gene (-a/aa). Hemoglobin electrophoresis is normal in adults with the alpha thalassemia carrier
state. Confirmation of the diagnosis requires molecular analysis. The condition is also referred
to as alpha thalassemia minima.
15) A 23-year-old female with sickle cell disease, who is 12 weeks pregnant comes to the
emergency department for severe pain in her upper and lower extremities along with pain in her
back. She denies fever, chills or dyspnea. In a year she has around 1 or 2 episodes of painful
crisis which she manages at home. On physical examination, blood pressure is 110/70 mmHg,
respiratory rate 16/minute, pulse rate 80/minute, and temperature of 37 degrees. She is diffusely
tender and in obvious distress due to pain. Cardiopulmonary examination is normal. She has a
gravid uterus. Neurological examination is normal. Lab studies show a hemoglobin of 8 g/dl and
WBC of 7000/microliter. Chest x-ray and urinalysis are normal. What is the most appropriate
treatment?
a) Hydroxyurea
b) Meperidine
c) Ketorolac
d) Morphine
a) Hydroxyurea
b) Meperidine
c) Ketorolac
d) Morphine
 Hydroxyurea is contraindicated in the first trimester of pregnancy. It should be stopped at least
three months before conception. Meperidine should not be used due to the risk of causing
seizures. Ketorolac is a pregnancy class C drug and is teratogenic. Morphine is the treatment
of choice for initial and maintenance pain management in all sickle cell painful crisis patients
including pregnant women.
PREGNANCY AND SICKLE
CELL DISEASE
 Vaso-occlusive crises are more common in pregnancy.
 SCD is NOT a contraindications to vaginal delivery.
 Hormonal contraceptives and copper-releasing intrauterine device are safe and effective
methods for women with SCD
16) A 58-year-old male develops marked onset of chest pain. He presents to the hospital and is
found to have an ST elevation MI. He is placed on nitroglycerin, aspirin, metoprolol and
clopidogrel. He undergoes a percutaneous coronary intervention with a metal stent deployed in
the right coronary artery.
Prior to hospital discharge, he was given Bactrim x 3 days for a UTI. Two weeks later, he has a
routine CBC with Plt drawn. His Platelets have dropped from 280,000/mm3 on his initial
hospital presentation down to 110,000/mm3. His Hb has dropped from 13.2 g/dl to 8.6 g/dl. His
PT/PTT/Fibrinogen are all WNL. A blood smear is reviewed and showed schistocytes. Which
of the following is the most likely offending agent?
A. Bactrim
B. Metoprolol
C. Aspirin
D. Clopidogrel
E. Stent induced hemolysis
16) A 58-year-old male develops marked onset of chest pain. He presents to the hospital and is
found to have an ST elevation MI. He is placed on nitroglycerin, aspirin, metoprolol and
clopidogrel. He undergoes a percutaneous coronary intervention with a metal stent deployed in
the right coronary artery.
Prior to hospital discharge, he was given Bactrim x 3 days for a UTI. Two weeks later, he has a
routine CBC with Plt drawn. His Platelets have dropped from 280,000/mm3 on his initial
hospital presentation down to 110,000/mm3. His Hb has dropped from 13.2 g/dl to 8.6 g/dl. His
PT/PTT/Fibrinogen are all WNL. A blood smear is reviewed and showed schistocytes. Which
of the following is the most likely offending agent?
A. Bactrim
B. Metoprolol
C. Aspirin
D. Clopidogrel
E. Stent induced hemolysis
Correct Option: D
D) Clopidogrel
Clopidogrel is a thienopyridine that may cause Thrombotic thrombocytopenic purpura (TTP).
TTP can be congenital or acquired deficiency of ADAMTS13 level.
Most thienopyridine-associated TTP cases are associated with auto-antibodies that inhibit the plasma
metalloprotease, ADAMTS13.
 
MANIFESTATIONS OF TTP
 Scenario: Inciting factor (pregnancy, clopidogrel, surgery) followed by drop in platelet count
and schistocytes in peripheral blood morphology.

 To confirm the diagnosis you need to measure ADAMTS13 level.

 Treatment: plasma exchange +/- steroid & Rituximab.

 > Caplacizumab which is an anti-VWF has been recently approved for treatment of TTP.
17) A 51-year-old man presents to the hospital complaining of abdominal pain and distension for a
few weeks. He further reports severe loss of libido and erectile dysfunction for the past six months.
On examination, he appears to be tanned, and there is mild peripheral edema and increased
abdominal circumference; however, the remainder of the examination is unremarkable. His past
medical history is significant for NSAIDs use for the relief of arthralgias. His laboratory
investigations reveal a gamma-glutamyl transferase of 55 U/L (normal range 9–48 U/L) and
transferrin saturation of 95%; serology shows positive HBeAb. What is the most appropriate
treatment for this patient?

a) Ursodeoxycholic acid
b) Phlebotomy
c) Desferrioxamine
d) Penicillamine
17) A 51-year-old man presents to the hospital complaining of abdominal pain and distension for a
few weeks. He further reports severe loss of libido and erectile dysfunction for the past six months.
On examination, he appears to be tanned, and there is mild peripheral edema and increased
abdominal circumference; however, the remainder of the examination is unremarkable. His past
medical history is significant for NSAIDs use for the relief of arthralgias. His laboratory
investigations reveal a gamma-glutamyl transferase of 55 U/L (normal range 9–48 U/L) and
transferrin saturation of 95%; serology shows positive HBeAb. What is the most appropriate
treatment for this patient?

a) Ursodeoxycholic acid
b) Phlebotomy
c) Desferrioxamine
d) Penicillamine
HEMOCHROMATOSIS
 Autosomoal recessive disorder.
 C282Y (account for >90% of the cases) or H63D mutation.
 Typical scenario: Iron overload manifestation in men >40 or postmenopausal women.
 Liver: Transamnitis, cirrhosis or HCC.
 Heart: Cardiomyopathy, diastolic dysfunction or conduction abnormalities.
 Endocrine ( DM “bronze diabetes”) , hypogonadism.
 Joints: Second and third MCP joint pain.
 Transferrin saturation is typically >45%.
 The history, physical exam, and lab findings are suggestive of hemochromatosis. Cirrhosis is
present in 70% of patients with hemochromatosis. In these patients, there is a marked
increased incidence of hepatocellular carcinoma, which is a major cause of death. If the
ferritin level is above 200 mcg/L in women or 300 mcg/L in men, or a transferrin saturation is
more than 40% in women or 50% in men it should lead to further testing. The conventional
therapy for primary hemochromatosis is phlebotomy. By drawing off red blood cells, the
major mobilizer of iron in the body, iron toxicity, can be minimized. Patients may require 50 to
100 phlebotomies of 500 mL each to reduce iron levels to normal. Phlebotomy is usually
performed once or twice a week. Once iron levels have normalized, lifelong, but less frequent,
phlebotomy (typically 3-4 times a year) is required. The objective is to obtain a ferritin level of
less than 50 mcg/L. Alcohol should be strictly prohibited in this condition because it can
accelerate liver and pancreatic toxicity. Preexisting end-organ damage is rarely reversed by
phlebotomy.
 18) A 45-year-old male patient presents to the hospital with complaints of fatigue, loss of libido,
change in skin color, and arthralgias for a few months. He has been suffering from diabetes
mellitus for six months. On examination, he appears to be tanned, and there is the loss of hair on
several parts of his body, particularly involving the pubic area. On chest auscultation, there is
fourth heart sound. Echocardiography is suggestive of restrictive cardiomyopathy. He is
particularly at risk of developing which of the following infections?
a) Klebsiella pneumonia
b) Salmonella.
c) Yersinia enterocolitica
d) Histoplasmosis
e) Human immunodeficiency virus (HIV)
a) Klebsiella pneumonia
b) Salmonella.
c) Yersinia enterocolitica
d) Histoplasmosis
e) Human immunodeficiency virus (HIV)
 Iron overload of macrophages can cause impaired phagocytosis and lead to decreased
immunity, resulting in an increased risk of infection from Listeria, Yersinia enterocolitica, and
Vibrio vulnificus.
 The conventional therapy for primary hemochromatosis is phlebotomy. Patients may require
50 to 100 phlebotomies of 500 mL each to reduce iron levels to normal. Phlebotomy is usually
performed once or twice a week. Once iron levels have normalized, lifelong, but less frequent,
phlebotomy (typically 3-4 times a year) is required. The objective is to obtain a ferritin level of
less than 50 mcg/L.
19) A 49-year-old male presents with weight loss, abdominal pain, generalized weakness,
arthritis, and erectile dysfunction. He has a history of diabetes mellitus and hypertension.
Physical exam shows hepatomegaly, gynecomastia, and testicular atrophy. The skin is slate gray,
and the joints show slight swelling of the third and fourth metacarpal phalangeal joints on the
left hand. Laboratory data shows transferrin saturation of 80% and ferritin of 7000
micrograms/L. Patients with this condition are advised to avoid which one of the following?
 
a) Shellfish
b) Chicken
c) Fresh vegetables
d) Beef
19) A 49-year-old male presents with weight loss, abdominal pain, generalized weakness,
arthritis, and erectile dysfunction. He has a history of diabetes mellitus and hypertension.
Physical exam shows hepatomegaly, gynecomastia, and testicular atrophy. The skin is slate gray,
and the joints show slight swelling of the third and fourth metacarpal phalangeal joints on the
left hand. Laboratory data shows transferrin saturation of 80% and ferritin of 7000
micrograms/L. Patients with this condition are advised to avoid which one of the following?
 
a) Shellfish
b) Chicken
c) Fresh vegetables
d) Beef
 Iron overload of macrophages can cause impaired phagocytosis and lead to decreased
immunity, resulting in increased risk of infection from Listeria, Yersinia enterocolitica, and
Vibrio vulnificus. Vibrio vulnificus can be acquired by eating raw or undercooked shellfish,
especially oysters, raw seafood.
 It can also cause wound infections after exposure of skin wounds to seawater.
20) A 47-year-old male presents to a gastroenterology clinic who has been referred by his
general practitioner with abnormal liver function tests. His symptoms are nausea, lethargy,
difficulty in breathing, loss of libido and generalized body aches. He is also suffering from
diabetes mellitus and congestive cardiac failure. His clinical examination reveals finger
clubbing, spider naevi, hyperpigmented skin, and tenderness in the right upper abdominal
quadrant. Which of the following is a reversible complication of this patient's disease?

a) Diabetes mellitus
b) Liver cirrhosis
c) Hypogonadotropic hypogonadism
d) Cardiomyopathy
e) Arthropathy.
20) A 47-year-old male presents to a gastroenterology clinic who has been referred by his
general practitioner with abnormal liver function tests. His symptoms are nausea, lethargy,
difficulty in breathing, loss of libido and generalized body aches. He is also suffering from
diabetes mellitus and congestive cardiac failure. His clinical examination reveals finger
clubbing, spider naevi, hyperpigmented skin, and tenderness in the right upper abdominal
quadrant. Which of the following is a reversible complication of this patient's disease?

a) Diabetes mellitus
b) Liver cirrhosis
c) Hypogonadotropic hypogonadism
d) Cardiomyopathy
e) Arthropathy.
 The patient is suffering from hemochromatosis which is caused by dysregulated iron
homeostasis and it leads to excessive iron absorption from the small intestine.
Cardiomyopathy is one of its reversible complications. The excessive deposition of iron in
different organs leads to various reversible and irreversible complications. Cardiomyopathy
and skin pigmentation are reversible complications. Diabetes mellitus,arthropathy, liver
cirrhosis, and hypogonadotropic hypogonadism are irreversible complications of
hemochromatosis.
21) A 65-year-old male with history of polycythemia vera who stopped phlebotomy 1 year ago,
presented to your clinic with fatigue and generalized weakness for the last 4 months. Physical
examination was notable for splenomegaly. Lab shows WBC 2,000, Hb 78, platelet 67,000.
Peripheral blood smear tear drop cells as demonstrated below. What is the most likely diagnosis:

a) Progression to acute myeloid leukemia


b) Refractory polycythemia vera
c) Myelofibrosis.
d) Aplastic anemia.
21) A 65-year-old male with history of polycythemia vera who stopped phlebotomy 1 year ago,
presented to your clinic with fatigue and generalized weakness for the last 4 months. Physical
examination was notable for splenomegaly. Lab shows WBC 2,000, Hb 78, platelet 67,000.
Peripheral blood smear tear drop cells as demonstrated below. What is the most likely diagnosis:

a) Progression to acute myeloid leukemia


b) Refractory polycythemia vera
c) Myelofibrosis.
d) Aplastic anemia.
 Whenever your hear : Teardrop-shaped red blood cells (leucoerythroblastic finding) think of
myelofibrosis.
 Myelofibrosis occurs when there is fibrosis of bone marrow which results in nucleated
erythrocytes and granulocytes in peripheral blood.
 Myelofibrosis can be primary or secondary ( secondary to polycythemia vera or essential
thrombocytosis).
 Polycythemia is one of myeloproliferative diseases that is characterized by high Hb and JAK2
mutation. It is typically treated with phlebotomy (Target hematocrit of <45).
 Loss of phlebotomy requirement and development of progressive anemia is a clue to
secondary myelofibrosis.
22) A 65-year-old male with a 2-year history of essential thrombocytosis presents to the clinic
for follow up. He has been on hydroxyurea for the past 1 year, however, his platelets numbers
are still well above 600000. He is concerned about himself as he takes his medication regularly.
He would like to know if there us other medications that could help reduce his platelet's
numbers. Which of the following is another alternative medication to help manage this patient's
essential thrombocytosis?
 
a) Aspirin
b) Doxorubicin
c) Anagrelide
d) There are no other medications with proven benefit
22) A 65-year-old male with a 2-year history of essential thrombocytosis presents to the clinic
for follow up. He has been on hydroxyurea for the past 1 year, however, his platelets numbers
are still well above 600000. He is concerned about himself as he takes his medication regularly.
He would like to know if there us other medications that could help reduce his platelet's
numbers. Which of the following is another alternative medication to help manage this patient's
essential thrombocytosis?
 
a) Aspirin
b) Doxorubicin
c) Anagrelide
d) There are no other medications with proven benefit
ESSENTIAL
THROMBOCYTOSIS
 It is a myeloproliferative neoplasm characterized by excessive platelent production with a
tendency for thrombosis and hemorrhage.
 In 90% of cases, a somatic mutation in either JAK2, CALR or MPL is found.
 Most cases are discovered incidentally (Thrombocytosis with no apparent cause). Some
patients may have complications such as headache, dizziness or thrombosis.
 Anagrelide is an alternative medication to help reduce the number of platelets in patients with
essential thrombocytosis. Anagrelide is associated with more side effects and typically
reserved for patients who are not on Hydroxyurea. Hydroxyurea is the first line and should be
used before trying anagrelide. Another known alternative is interferon.
23) A 65-year-old man presents to the clinic with recently diagnosed polycythemia vera. Six
months prior to his diagnosis, he reported experiencing burning pain in the bilateral lower
extremities relieved only by dunking them in cold water. What is the most likely underlying
pathology?
 
a) Erythromelalgia
b) Diabetic neuropathy
c) Peripheral arterial disease
d) Capillary thrombosis
23) A 65-year-old man presents to the clinic with recently diagnosed polycythemia vera. Six
months prior to his diagnosis, he reported experiencing burning pain in the bilateral lower
extremities relieved only by dunking them in cold water. What is the most likely underlying
pathology?
 
a) Erythromelalgia
b) Diabetic neuropathy
c) Peripheral arterial disease
d) Capillary thrombosis
Erythromelalgia is a rarely occurring disease entity characterized by a triad of erythema,
warmth, and recurrent burning pain, most notably affecting the extremities. Erythromelalgia can
be classified as being either primary or secondary. Patients will frequently present with reports
of red, hot, swollen acral sites including the hands, feet, and rarely the face. The pain is often
described as a burning, searing pain that is triggered by exercise, warm climates, and wearing
tight-fitting shoes amongst other inciting agents. Often times, erythromelalgia can be the
presenting manifestation of secondary causes.
24) A 52-year-old female is scheduled to undergo an elective hysterectomy for several uterine
fibroids. She has never had any surgery before and has no significant past medical history. Labs
shows: PT=11 seconds (Normal),PTT=80 seconds (elevated), Fibrinogen=Normal, Her CBC with
Plt/Diff are WNL.
No history of bleeding or clotting and she is not on any medication. Further workup is needed of her
elevated PTT. Which of the following tests would be most helpful to order at this point?
A. Clot lysis testing
B. Factor VII levels
C. Platelet aggregation testing
D. Mixing study
E. review peripheral blood smear
24) A 52-year-old female is scheduled to undergo an elective hysterectomy for several uterine
fibroids. She has never had any surgery before and has no significant past medical history. Labs
shows: PT=11 seconds (Normal),PTT=80 seconds (elevated), Fibrinogen=Normal, Her CBC with
Plt/Diff are WNL.
No history of bleeding or clotting and she is not on any medication. Further workup is needed of her
elevated PTT. Which of the following tests would be most helpful to order at this point?
A. Clot lysis testing
B. Factor VII levels
C. Platelet aggregation testing
D. Mixing study
E. review peripheral blood smear
Correct Option: D
D) Mixing study
This patient has an elevation of PTT, but a normal PT
and Fibrinogen.
A mixing study at this juncture will help delineate the
difference between this patient have a factor deficiency
or an inhibitor.
In a Mixing study, normal plasma is mixed with patient
plasma (1:1 ratio) and is incubated for 2 hours at 37
degrees. If there is correction in the PT or PTT value, the
patient will be deemed to have a factor deficiency. If the
PT or PTT does not correct, the patient will have an
underlying inhibitor.
25) A 14-year-old male with a history of mild hemophilia A presents for evaluation of bleeding
from a right knee abrasion after an inadvertent fall. Which of the following is the best initial
therapy for this patient?
 
a) Tranexamic acid
b) Epsilon-aminocaproic acid
c) Desmopressin
d) Topical thrombin
25) A 14-year-old male with a history of mild hemophilia A presents for evaluation of bleeding
from a right knee abrasion after an inadvertent fall. Which of the following is the best initial
therapy for this patient?
 
a) Tranexamic acid
b) Epsilon-aminocaproic acid
c) Desmopressin
d) Topical thrombin
 Patients with mild hemophilia A with documented prior responses to desmopressin may be
administered desmopressin, which promotes the release of endogenous factor VIII, for
treatment of minor bleeding. Patients with mild hemophilia A can receive desmopressin to
treat minor bleeding if they have been previously been documented to respond to
desmopressin in order to ensure an adequate hemostatic response.
 Patients with mild hemophilia A who have significant bleeding should receive factor VIII
replacement therapy just as those with moderate or severe factor VIII do. Antifibrinolytic
agents such as tranexamic acid and epsilon-aminocaproic acid may be administered as single
agents with mild mucosal bleeding or in combination with factor VIII replacement therapy for
individuals with hemophilia A with significant mucosal bleeding.
26) A 14-year-old male with hemophilia A is undergoing surgery to remove his tonsils. Which of
the following is the best option for preventing bleeding complications?
 
a) Fresh frozen plasma and cryoprecipitate
b) Desmopressin
c) Erythropoietin
d) Recombinant factor VIII
26) A 14-year-old male with hemophilia A is undergoing surgery to remove his tonsils. Which of
the following is the best option for preventing bleeding complications?
 
a) Fresh frozen plasma and cryoprecipitate
b) Desmopressin
c) Erythropoietin
d) Recombinant factor VIII
 Hemophilia A is a coagulation disorder resulting from a deficiency or abnormality of factor
VIII. Although fresh frozen plasma contains factor VIII, the levels are too low to prevent or
control bleeding in hemophilia. Desmopressin is a synthetic analog of an anti-diuretic
hormone that increases levels of factor VIII and von Willebrand factor. Desmopressin can be
used alone for mild hemophilia A, but it is ineffective in severe forms of the disease.
Erythropoietin does not prevent bleeding complications in hemophilia. Recombinant factor
VIII is given prophylactically prior to surgery.
27) A 55-year-old man with a history of Crohn disease, pernicious anemia, and diabetes mellitus
has been hospitalized for an exacerbation of Crohn disease. During the hospitalization, he
develops multiple ecchymoses and bleeding from IV insertion sites. The patient reports no
family history of any bleeding disorders. The patient is not taking any anticoagulation or
antiplatelets. Blood work shows hemoglobin 10.2 g/dL, platelet count 220,000/microL, PT 12.5
(N), PTT 52 (high), and normal fibrinogen and INR. PTT after mixing study 50. What is the
most likely cause of the patient's condition?
 
a) Disseminated intravascular coagulation
b) Acquired hemophilia
c) Scurvy
d) Von Willebrand factor deficiency
27) A 55-year-old man with a history of Crohn disease, pernicious anemia, and diabetes mellitus
has been hospitalized for an exacerbation of Crohn disease. During the hospitalization, he
develops multiple ecchymoses and bleeding from IV insertion sites. The patient reports no
family history of any bleeding disorders. The patient is not taking any anticoagulation or
antiplatelets. Blood work shows hemoglobin 10.2 g/dL, platelet count 220,000/microL, PT 12.5
(N), PTT 52 (high), and normal fibrinogen and INR. PTT after mixing study 50. What is the
most likely cause of the patient's condition?
 
a) Disseminated intravascular coagulation
b) Acquired hemophilia
c) Scurvy
d) Von Willebrand factor deficiency
Acquired hemophilia is a rare autoimmune disorder that occurs in patients with no family
history of bleeding disorders or hemorrhage due to autoantibodies developing against
coagulation factors.
Patients with acquired hemophilia have a pre-existing underlying autoimmune condition,
malignancy, and infections like HIV and Hepatitis (B and C). This patient with hemophilia has a
prolonged activated partial thromboplastin time (aPTT). However, a normal aPTT does not
exclude hemophilia and a normal PT (prothrombin time), complete blood count could show
normal or low hemoglobin level with normal platelet level.
Hemophilia is an inherited condition caused by the deficiency of one of the clotting factors in
the blood. It is almost always due to a defect or mutation in the gene for the clotting factor. The
encoding genes for factors VIII and factor IX are present in chromosome X. Factor IX is
synthesized by the hepatocytes and considered to be part of the intrinsic pathway. Its deficiency
will result in the defective coagulation cascade and insufficient fibrin mesh formation.
• Intrinsic pathway is measured by PTT.
• Extrinsic pathway is measured by PT.
• Deficiency in factor VII causes isolated
PT prolongation.
• Deficiency in XI, IX, VIII causes
prolonged PTT.
• Deficiency in V, X, Thrombin and Fibrin
causes prolongation of both PT & PTT.

• Coagulation studies measure the time it


takes for coagulation pathway to reach
fibrin>> this is why factor XIII deficiency
does NOT cause prolongation of PT or
PTT.

• Mixing studies help to rule out the


presence of inhibitors.
28) A conservationist ventured to live in a jungle for 5 months but had to cut his experiment
short when he began to suffer from bleeding gums, some teeth falling out, and red spots on the
thighs and legs. This person is suffering from a failure to properly synthesize which one of the
following proteins?
 
a) Myoglobin
b) Collagen
c) Insulin
d) Fibrillin
The conservationist is suffering from scurvy, a deficiency of vitamin C. In the synthesis of collagen
the hydroxylation of proline and lysine residues requires vitamin C and oxygen.
In the absence of Vitamin C, the collagen formed cannot be stabilized and is easily degraded leading
to the symptoms seen in this patient. The vitamin C pool in the body is usually depleted in 4-12
weeks if one stops the intake of the vitamin.
The long duration of stay by the conservationist in the jungle caused the deficiency. Dermatologic
findings include poor wound healing, gingival swelling with loss of teeth, mucocutaneous petechiae,
ecchymosis, and hyperkeratosis. Endochondral bone formation also is involved and patients may
have long bone atrophy or a “scorbutic rosary.”
Rheumatologic problems occur, including painful hemarthrosis and subperiosteal hemorrhage.
Alopecia is common and bleeding into the joints occurs with minimal trauma. Follicular
hyperkeratosis, hemolytic anemia, anxiety disorder, hysteria, depression, and fatigue are also
common.
29) A 53-year-old man with a past medical history of systemic lupus erythematosus is admitted
for elective cholecystectomy. The pre-operative anesthesia check to assess for fitness for surgery
was unremarkable. He undergoes laparoscopic cholecystectomy, which goes by uneventfully.
Twenty-four hours later, the patient develops hypotension and tachycardia in the post-operative
ward. He is found to have severe anemia. Further imaging reveals a large retroperitoneal
hematoma. PT and PTT are normal. The platelet counts, morphology of platelets, and thrombin
time is normal. Fibrinogen level and activity are normal. The family denies any history of
bleeding disorder in the patient or the family. The patient is taken to the operating room for re-
exploring the abdomen to detect any sites of bleeding. His cholecystectomy stump is noted to be
oozing. The bleeding is controlled with characterization. However, the patient bleeds again after
24 hours. What is the next best step in management?
a) Check factor VIII level and give factor VIII replacement.
b) Check factor IX level and give factor IX replacement.
c) Factor XIII deficiency test and give factor XIII replacement.
d) Give tranexamic acid.
a) Check factor VIII level and give factor VIII replacement.
b) Check factor IX level and give factor IX replacement.
c) Factor XIII deficiency test and give factor XIII replacement.
d) Give tranexamic acid.
This patient likely has acquired factor XIII
deficiency. A normal PT, aPTT, thrombin time,
and fibrinogen levels should point towards a
factor deficiency or defect which comes after
fibrin formation. Quantitative tests are now the
recommended tests to screen for factor XIII
deficiency.
Treatment is with factor XIII replacement or
FFP
30) A 35-year-old male presents to the clinic with a chief complaint of bleeding gums while
brushing teeth for the last 3 months.. He also reports occasional heartburn and a rash on his legs.
On physical exam, he is noted to have petechiae on bilateral ankles. Laboratory workup shows a
hemoglobin of 14.5 gm/dL, a white blood cell count of 4500/microL, and a platelet count of
50,000/microL. The coagulation profile and peripheral blood smear are normal. Which of the
following is the next best step in the management of this patient?
 
a) Platelet transfusion
b) Antiplatelet antibody testing.
c) Testing for Helicobacter pylori infection
d) Bone marrow biopsy
30) A 35-year-old male presents to the clinic with a chief complaint of bleeding gums while
brushing teeth for the last 3 months.. He also reports occasional heartburn and a rash on his legs.
On physical exam, he is noted to have petechiae on bilateral ankles. Laboratory workup shows a
hemoglobin of 14.5 gm/dL, a white blood cell count of 4500/microL, and a platelet count of
50,000/microL. The coagulation profile and peripheral blood smear are normal. Which of the
following is the next best step in the management of this patient?
 
a) Platelet transfusion
b) Antiplatelet antibody testing.
c) Testing for Helicobacter pylori infection
d) Bone marrow biopsy
 Thrombocytopenia with no evident cause = ITP.
 Patients with ITP and GI symptoms should be tested for H.pylori, which is associated with the
disease.
 Bone marrow biopsy is indicated only in patients with other hematological abnormalities or in
patients who didn’t respond to initial therapies.
 Antiplatelet antibody testing has low sensitivity and doesn’t correlate with clinic outcome. This
choice is almost always WRONG.
31) A 56-year-old female presents to clinic with a platelet count of 19,000/microL. She has an
exhaustive workup performed. After everything is ruled out, she is diagnosed with Immune
thrombocytopenic purpura.
 
Looking back at her platelet values, her normal platelet baseline for the past 5 years has ranged
between 32,000/microL to 47,000/microL. She has some increasing purpura on her body and
along with some GI bleeding felt to be due to a hemorrhoid. What do you recommend at this
time?
A. Observation
B. Steroids
C. Splenectomy
D. Rituximab
E. Eltrombopag
31) A 56-year-old female presents to clinic with a platelet count of 19,000/microL. She has an
exhaustive workup performed. After everything is ruled out, she is diagnosed with Immune
thrombocytopenic purpura.
 
Looking back at her platelet values, her normal platelet baseline for the past 5 years has ranged
between 32,000/microL to 47,000/microL. She has some increasing purpura on her body and
along with some GI bleeding felt to be due to a hemorrhoid. What do you recommend at this
time?
A. Observation
B. Steroids
C. Splenectomy
D. Rituximab
E. Eltrombopag
Correct Option: B
B) Steroids
 
One should not treat a patient with ITP unless there are signs of active bleeding or if there is a very
low platelet count. Many patients have no clinical issues with moderate thrombocytopenia and do
not require any form of therapy.
 
32) A 62-year-old female with known Hepatitis C with cirrhosis presents to her Primary
care physician for severe right frontal headaches. Brain MRI is performed that reveals a
right frontal mass that is very concerning for a glioma. She is evaluated by Neurosurgery
and is scheduled for surgery for both diagnostic/therapeutic purposes.
Baseline labs reveals normal coagulation studies. However, her platelet count is
48,000/mm3. The latter is due to hypersplenism and represents her baseline platelet value.
The patient will be receiving a platelet transfusion prior to surgery. What should be the goal
platelet level prior to her neurosurgery?
A. 50,000/mm3
B. 60,000/mm3
C. 75,000/mm3
D. 100,000/mm3
E. 150,000/mm3
32) A 62-year-old female with known Hepatitis C with cirrhosis presents to her Primary
care physician for severe right frontal headaches. Brain MRI is performed that reveals a
right frontal mass that is very concerning for a glioma. She is evaluated by Neurosurgery
and is scheduled for surgery for both diagnostic/therapeutic purposes.
Baseline labs reveals normal coagulation studies. However, her platelet count is
48,000/mm3. The latter is due to hypersplenism and represents her baseline platelet value.
The patient will be receiving a platelet transfusion prior to surgery. What should be the goal
platelet level prior to her neurosurgery?
A. 50,000/mm3
B. 60,000/mm3
C. 75,000/mm3
D. 100,000/mm3
E. 150,000/mm3
Correct Option: D
D) 100,000/mm3
Before any patient undergoes surgery, the platelet count (along with the coagulation studies in some
instances), should be known to see if a platelet transfusion is needed.
 
 Thresholds and situations to consider platelet transfusion for an invasive procedure:

<10,000/mm3 in an uncomplicated patient


<20,000/mm3 if a patient is undergoing a central line placement
<50,000/mm3 if a patient is undergoing major surgery
<50,000/mm3 if a patient is undergoing an endoscopic procedure for therapeutic purposes
<80,000/mm3 is a patient is undergoing epidural anesthesia
<100,000/mm3 if a patient is undergoing any neurosurgery or ophthalmology procedures
 For patients who are actively bleeding and have thrombocytopenia, they should be transfused to
keep the platelet count >50,000/mm3 or >100,000/mm3 if there is CNS bleeding.
33) A 32-year-old male presents to Infectious Disease clinic for recently diagnosed HIV. He thinks he
contracted the infection from a tattoo he received 2 years ago. His CD4 is around 250/mm3 and the
viral load is 65,000 copies/mL.
Prior to starting HAART therapy, his baseline labs are drawn. The Platelet count is 48,000/mm3. An
extensive workup is performed and it is felt that the patient has HIV-induced ITP. What therapy do
you offer at this time?
A. Steroids
B. IVIG
C. Splenectomy
D. HAART therapy
E. Rituximab
33) A 32-year-old male presents to Infectious Disease clinic for recently diagnosed HIV. He thinks he
contracted the infection from a tattoo he received 2 years ago. His CD4 is around 250/mm3 and the
viral load is 65,000 copies/mL.
Prior to starting HAART therapy, his baseline labs are drawn. The Platelet count is 48,000/mm3. An
extensive workup is performed and it is felt that the patient has HIV-induced ITP. What therapy do
you offer at this time?
A. Steroids
B. IVIG
C. Splenectomy
D. HAART therapy
E. Rituximab
Correct Option: D
D) HAART therapy
HIV-associated ITP clinically resembles classic ITP. However, patients with HIV associated ITP normally have
less bleeding.
 
In general, for patients who have not received HAART therapy yet and have moderate thrombocytopenia, defined
as a platelet count > 40,000 to 50,000/mm3, initiation of HAART alone is sufficient to improve the platelet count.
These patients do not need to be started on ITP type of medications such as steroids. Now, if after initiation of
HAART therapy, the platelet count remains low, then other secondary medications can be given including steroids.
 
While treatment for HIV-associated ITP is the same as that given for classic ITP (Steroids, IVIG etc.), HAART
therapy is quite effective in reversing the thrombocytopenia. In addition, need to be very careful giving
immunosuppressive medications to a patient with HIV ITP. This could potentiate development of opportunistic
infections like PJP.
34) A 28-year-old female is currently pregnant at 26 weeks gestation. This is her first
pregnancy. Her blood pressure remained persistently elevated (160’s/90’s) for the past 6
weeks and she has been having her blood pressure measured weekly.
A 24-hour urine testing is performed that reveals 1 gram protein/24 hours. She has
thrombocytopenia 110,000/mm3. Blood smear shows 1 schistocyte. PT/PTT/Fibrinogen and
LFTs are WNL. What is the most likely condition that she has?
A. HELLP
B. HUS
C. TTP
D. AFLP (acute fatty liver of pregnancy)
E. Preeclampsia
34) A 28-year-old female is currently pregnant at 26 weeks gestation. This is her first
pregnancy. Her blood pressure remained persistently elevated (160’s/90’s) for the past 6
weeks and she has been having her blood pressure measured weekly.
A 24-hour urine testing is performed that reveals 1 gram protein/24 hours. She has
thrombocytopenia 110,000/mm3. Blood smear shows 1 schistocyte. PT/PTT/Fibrinogen and
LFTs are WNL. What is the most likely condition that she has?
A. HELLP
B. HUS
C. TTP
D. AFLP (acute fatty liver of pregnancy)
E. Preeclampsia
Correct Option: E
E) Preeclampsia.
This patient has mild thrombocytopenia, mild hemolysis, no coagulation problems and severe HTN.
There is no evidence of liver problems. Severe proteinuria is defined ≥ 1 gram/24 hours. None of the
other choices are associated with proteinuria
 > For both AFLP and HELLP, one would be expected to see liver abnormalities.  
> The following define pre-eclampsia:
- Onset at >20 weeks’ gestational age of 24-hour proteinuria ≥ 30 mg/day
- Systolic blood pressure >140 mmHg or diastolic blood pressure ≥ 90 mmHg as measured twice, 4–6
hours and less than 7 days apart.
 - One should see disappearance of all these abnormalities before the end of the 6th week postpartum.
 
 
Delivery is the only curative treatment for pre-eclampsia. However, patients with preeclampsia
without severe disease can be monitored with delivery after 37 weeks.
Antihypertensive therapy is indicated for treatment of persistent severe hypertension (defined as
systolic blood pressure ≥ 160 mmHg and diastolic blood pressure ≥ 110 mmHg) to prevent a stroke.
 
For women with preeclampsia and features of severe disease, it is recommended to give intrapartum
and postpartum seizure prophylaxis.
 
  
SCENARIOS
 HELLP:

-  Patients with hemolysis, elevated liver enzymes and low platelets.


- Treatment includes expeditious delivery of the fetus and supportive care of the mother
  
 Acute fatty liver of pregnancy:  

-Occurs in the 3rd trimester


 - One sees RUQ pain, jaundice, and cholestatic liver dysfunction
- Signs of liver disease is more severe than HELLP (hypoglycemia, ascites, encephalopathy..etc).
 - Thrombocytopenia is usually mild.
Management: For AFLP, delivery of the fetus and supportive management of the mother
35) A 76-year-old woman with a past medical history of diabetes, hypertension, heart failure
with reduced ejection fraction, and end-stage renal disease on dialysis is admitted to the hospital
for community-acquired pneumonia. She is initially given subcutaneous heparin for DVT
prophylaxis. Her stay was complicated with bilateral deep vein thrombosis, a fall in platelets,
and a positive serotonin release assay. Heparin is stopped, and she is prescribed argatroban
infusion. Which of the following, if any, is the most appropriate strategy to monitor the
therapeutic efficacy of argatroban in this patient?
a) Anti-Xa factor
b) aPTT
c) INR
d) Monitoring is not indicated
35) A 76-year-old woman with a past medical history of diabetes, hypertension, heart failure
with reduced ejection fraction, and end-stage renal disease on dialysis is admitted to the hospital
for community-acquired pneumonia. She is initially given subcutaneous heparin for DVT
prophylaxis. Her stay was complicated with bilateral deep vein thrombosis, a fall in platelets,
and a positive serotonin release assay. Heparin is stopped, and she is prescribed argatroban
infusion. Which of the following, if any, is the most appropriate strategy to monitor the
therapeutic efficacy of argatroban in this patient?
a) Anti-Xa factor
b) aPTT
c) INR
d) Monitoring is not indicated
Argatroban is a direct thrombin inhibitor; it can inhibit both free and clot-bound thrombin, and it
also inhibits thrombin-catalyzed reactions, mainly in the intrinsic pathway, leading to aPTT
prolongation. According to the argatroban package insert, therapy is monitored using the aPTT
with a target range of 1.5 to 3 times the initial baseline value (not to exceed 100 seconds).
Argatroban falsely elevates INR, due to an interaction with the thromboplastin reagent.
Prothrombin time (PT) and thrombin time (TT) are affected by argatroban as well; however, the
therapeutic ranges for these tests have not been identified for argatroban therapy and should not
be used for argatroban dose adjustments. Argatroban is a direct thrombin inhibitor. It does not
affect the anti-Xa assay. Anti-Xa is not an appropriate monitoring parameter.
36) A 70-year-old female came to the emergency department with sudden onset right-side
weakness. Initial CT scan on arrival showed a 4 cm right basal ganglia hemorrhage. On obtaining
further history, the patient admits to taking apixaban for atrial fibrillation. Immediately the systolic
blood pressure is lowered to less than 160 mm Hg with nitroprusside to prevent further hematoma
expansion. Which of the following is the best immediate step in the management for this patient?
 
a) Vitamin K
b) Cryoprecipitate
c) Andexanet alfa
d) Idarucizumab
e) Dialysis
36) A 70-year-old female came to the emergency department with sudden onset right-side
weakness. Initial CT scan on arrival showed a 4 cm right basal ganglia hemorrhage. On obtaining
further history, the patient admits to taking apixaban for atrial fibrillation. Immediately the systolic
blood pressure is lowered to less than 160 mm Hg with nitroprusside to prevent further hematoma
expansion. Which of the following is the best immediate step in the management for this patient?
 
a) Vitamin K
b) Cryoprecipitate
c) Andexanet alfa
d) Idarucizumab
e) Dialysis
Vitamin K is useful in the reversal of warfarin-related bleeding. Cryoprecipitate is useful in the
reversal of tPA-related bleeding. It is not useful in apixaban related bleeding.
Andexanet alfa was developed for reversal of factor Xa inhibitors like apixaban and
rivaroxaban. Administration is usually as a bolus followed by a two-hour infusion. Almost 82%
of patients have excellent to good hemostatic efficacy at 12 hours.
The common side effects of andexanet are urinary tract infection and pneumonia.
Idarucizumab is a monoclonal antibody designed for the reversal of dabigatran.
Anticoagulant Antidote

Warfarin PCC + vitamin K is preferred. If not available,


use FFP
Rivaroxaban, Apixaban Andexanet alfa

Dabigatran Idarucizumab

Heparin/Enoxaparin Protamine sulfate

Fondaparinux Andexanet alfa (not FDA approved) or aPCC.


37) A 72-year-old male with a history of Type II DM and HTN has been following Cardiology for
management of his severe high-gradient Aortic stenosis. Recently, he has developed worsening Chest
pain and shortness of breast with minimal exertion. It is strongly recommended that he undergo a
mechanical valve replacement. In addition to aspirin, which of the following do you offer as post-
procedure anticoagulation?
A. Dabigatran
B. Rivaroxaban
C. Apixaban
D. Edoxaban
E. Heparin/Warfarin
37) A 72-year-old male with a history of Type II DM and HTN has been following Cardiology for
management of his severe high-gradient Aortic stenosis. Recently, he has developed worsening Chest
pain and shortness of breast with minimal exertion. It is strongly recommended that he undergo a
mechanical valve replacement. In addition to aspirin, which of the following do you offer as post-
procedure anticoagulation?
A. Dabigatran
B. Rivaroxaban
C. Apixaban
D. Edoxaban
E. Heparin/Warfarin
Correct Option: E) Heparin/Warfarin
Risk of systemic embolization following mechanical valve is 1% on warfarin, >2% on ASA and 4% with no
anticoagulation. Patients with a mitral valve prosthesis have twice the risk of this happening compared to those who
have a aortic valve prostheses.
Patients with mechanical prosthetic valves should be on long-term treatment with Warfarin and Aspirin (81 mg).
Heparin can be used as a bridge until a patient is therapeutic on Warfarin.
Direct oral anticoagulants (DOACs), including the oral direct thrombin inhibitor dabigatran and the oral direct
factor Xa inhibitors rivaroxaban, apixaban, edoxaban should not be used in patients with mechanical valves.
In the RE-ALIGN study, Dabigatran showed no benefit and an excess of thromboembolic events and bleeding risks
compared to warfarin for patients who underwent a mechanical valve replacement.
The same is true for DOAC and mechanical heart valve.
For patients who have a surgically placed bioprosthetic aortic or mitral valve, one should consider a baby aspirin
wand warfarin for the first 3-6 months and then aspirin thereafter.

 
38) The 52-year-old male is currently admitted to the hospital after having suffered an embolic stroke.
Neurology and Cardiology were consulted and after a thorough workup, no etiology for the stroke has been
found. A TEE is performed and is normal.
 
Hematology was consulted and they perform a serologic workup. Lupus anticoagulant was positive and
both anti-cardiolipin antibodies and Beta 2-glycoprotein antibodies were floridly positive. What
anticoagulant do you recommend for this patient?
A. Warfarin
B. Rivaroxaban
C. Apixaban
D. Edoxaban
E. None
38) The 52-year-old male is currently admitted to the hospital after having suffered an embolic stroke.
Neurology and Cardiology were consulted and after a thorough workup, no etiology for the stroke has been
found. A TEE is performed and is normal.
 
Hematology was consulted and they perform a serologic workup. Lupus anticoagulant was positive and
both anti-cardiolipin antibodies and Beta 2-glycoprotein antibodies were floridly positive. What
anticoagulant do you recommend for this patient?
A. Warfarin
B. Rivaroxaban
C. Apixaban
D. Edoxaban
E. None
Correct Option: A
A) Warfarin
 A group of investigators tested the efficacy and safety of rivaroxaban as compared to warfarin in high-risk patients
with thrombotic Antiphospholipid Syndrome.
A randomized, open-label, study evaluated Rivaroxaban 20 mg once daily was compared to warfarin (INR target
2.5), for the prevention of thromboembolic events, major bleeding and vascular death in patients with
Antiphospholipid Syndrome. 
The trial was terminated prematurely after the enrollment of 120 patients (59 randomized to rivaroxaban and 61 to
warfarin) because of an excess of events among patients in the rivaroxaban arm. Mean follow up was 569 days.

The use of rivaroxaban in high-risk patients with Antiphospholipid Syndrome was associated with an increased rate
of thrombosis as compared with warfarin, thus showing no benefit and excess risk.
39) A 58-year-old male presents with a recurrent right leg thrombosis. His first thrombotic event occurred 9 years
ago and occurred in the right leg. The etiology for this clot was uncertain. He completed 3 months of warfarin and
then stopped the treatment.
 
Recently, he developed acute onset right leg swelling. An ultrasound was performed that revealed an acute DVT.
He had dyspnea and a PE Protocol CT confirmed a right-sided pulmonary embolism. He has no underlying
medical conditions and he is up to date with his cancer screening. He is a never smoker. What do you
recommend?
a) IVC filter and enoxaparin
b) IVC filter and rivaroxaban
c) IVC filter and coumadin
d) Rivaroxaban
e) Streptokinase followed by Rivaroxaban
39) A 58-year-old male presents with a recurrent right leg thrombosis. His first thrombotic event occurred 9 years
ago and occurred in the right leg. The etiology for this clot was uncertain. He completed 3 months of warfarin and
then stopped the treatment.
 
Recently, he developed acute onset right leg swelling. An ultrasound was performed that revealed an acute DVT.
He had dyspnea and a PE Protocol CT confirmed a right-sided pulmonary embolism. He has no underlying
medical conditions and he is up to date with his cancer screening. He is a never smoker. What do you
recommend?
a) IVC filter and enoxaparin
b) IVC filter and rivaroxaban
c) IVC filter and coumadin
d) Rivaroxaban
e) Streptokinase followed by Rivaroxaban
Correct Option: D
D) Rivaroxaban
One should not use inferior vena cava (IVC) filters routinely in patients who have acute venous thrombosis.
Indications for IVC filters are:
Acute VTE with contraindication to anticoagulation (e.g. active bleeding).

Patients who develop a pulmonary embolism despite therapeutic anticoagulation,


 Patients with a massive PE and poor cardiopulmonary reserve.
 If a filter is needed, then a retrievable filter should be placed. The filter should be removed when the risk
for PE has resolved and/or when anticoagulation can be safely resumed.
The question described a patient with no contraindication to anticoagulation. The patient should be given
definitive anticoagulation without having an IVC filter placed.
40) A 68-year-old male with a past medical history significant for HTN and Type II DM has just
been diagnosed with a RLE DVT. His labs were sent for and are pending. What is the CrCl
whereby you should not administer Rivaroxaban to this patient?

A. CrCl <10 mL/min


B. CrCl <15 mL/min
C. CrCl <20 mL/min
D. CrCl <30 mL/min
E. CrCl <60 mL/min
40) A 68-year-old male with a past medical history significant for HTN and Type II DM has just
been diagnosed with a RLE DVT. His labs were sent for and are pending. What is the CrCl
whereby you should not administer Rivaroxaban to this patient?

A. CrCl <10 mL/min


B. CrCl <15 mL/min
C. CrCl <20 mL/min
D. CrCl <30 mL/min
E. CrCl <60 mL/min
Correct Option: D
D) CrCl <30 mL/min
Pharmacokinetic studies reveal that rivaroxaban exposure is increased up to 60% in patients who
have underlying renal impairment.
 For the treatment of DVT or PE or for VTE prophylaxis:

Avoid the use of Rivaroxaban in patients who have a CrCl < 30 mL/min
In the seminal EINSTEIN studies, in which oral rivaroxaban was studied in patients who had
VTE/PE, patients who had a CrCl <30 mL/min were excluded.
However, for nonvalvular atrial fibrillation, avoid the use of Rivaroxaban in patients with CrCl <15
mL/min.  
 
41) A 62-year-old male who is on warfarin for a saddle pulmonary embolism that was
diagnosed 2 months ago, presents to the ED after being involved in a recent car accident.
His undergoes an urgent CT scan of the Head that reveals bleeding in his brain. Labs are
sent for and his INR > 15. His wife mentions that he had been on levofloxacin for the past 2
weeks for a community acquired pneumonia. The trauma team is concerned that the patient
may expire given the nature of his intracranial bleeding. What treatment should be given?
A. Fresh Frozen Plasma and vitamin K.
B. cryoprecipitate
C. Protamine Sulfate
D. Prothrombin Complex concentrate and Vitamin K
E. factor VII (Novoseven).
41) A 62-year-old male who is on warfarin for a saddle pulmonary embolism that was
diagnosed 2 months ago, presents to the ED after being involved in a recent car accident.
His undergoes an urgent CT scan of the Head that reveals bleeding in his brain. Labs are
sent for and his INR > 15. His wife mentions that he had been on levofloxacin for the past 2
weeks for a community acquired pneumonia. The trauma team is concerned that the patient
may expire given the nature of his intracranial bleeding. What treatment should be given?
A. Fresh Frozen Plasma and vitamin K.
B. cryoprecipitate
C. Protamine Sulfate
D. Prothrombin Complex concentrate and Vitamin K
E. factor VII (Novoseven).
Correct Option: D
D) Prothrombin Complex concentrate (PCC) and Vitamin K.
PCC can reverse the effects of warfarin. The 4-factor PCC contains factors II, VII, IX, X, Protein C
and Protein S (these are the same factors inhibited by warfarin).
It is used in cases of significant bleeding in the setting of coagulopathy (INR > 8, elevation of PTT).
PCC restores the INR significantly faster than Fresh frozen plasma (FFP).  

 Factor VII (NovoSeven): used for hemophilia with inhibitor

 Cryoprecipitate : for Fibrinogen repletion (DIC)


42) A 42-year-old female presents to the hospital with a LLE DVT and PE. She is started on enoxaparin and
warfarin. Within 18 hours of starting her first dose of warfarin, she develops skin lesions throughout her body. It is
suspected that she has developed Warfarin skin necrosis and the drug is stopped immediately. There is no serologic
evidence for DIC.
She reports that several of her family members had blood clots. . She says that they were all diagnosed with a
hypercoagulable condition, but she is unclear which one. Which of the following hypercoagulable condition does
she likely have?
A. AT III deficiency
B. Factor V Leiden mutation (Heterozygous)
C. Prothrombin gene mutation
D. Antiphospholipid antibody syndrome
E. Protein C deficiency
42) A 42-year-old female presents to the hospital with a LLE DVT and PE. She is started on enoxaparin and
warfarin. Within 18 hours of starting her first dose of warfarin, she develops skin lesions throughout her body. It is
suspected that she has developed Warfarin skin necrosis and the drug is stopped immediately. There is no serologic
evidence for DIC.
She reports that several of her family members had blood clots. . She says that they were all diagnosed with a
hypercoagulable condition, but she is unclear which one. Which of the following hypercoagulable condition does
she likely have?
A. AT III deficiency
B. Factor V Leiden mutation (Heterozygous)
C. Prothrombin gene mutation
D. Antiphospholipid antibody syndrome
E. Protein C deficiency
Correct Option: E
E) Protein C deficiency
This patient likely has either heterozygous or homozygous protein C deficiency. Individuals with
hereditary protein C deficiency lack the natural anticoagulant function of activated protein C and are
at risk for clinical phenotypes associated with increased thrombotic risk including VTE, warfarin-
induced skin necrosis, and (in patients with homozygote disease) neonatal purpura fulminans.
Remember, that the half-lives of the vitamin K proteins differ. The half life of protein C is about 4-6
hours. In distinction, the half life of the other vitamin K coagulation proteins include Factor VII (3-5
hours), Factor IX (18-24 hours), Factor X (40-60 hours) and Factor II (72 -100 hours).
.
 
 
 
 
 Warfarin antagonizes protein C very quickly. As protein C, an endogenous anti-coagulant produced
by the liver that inactivates factors Va and VIIIa, is decreased most quickly by warfarin, the patient
can initially develop an underlying procoagulant state. For a patient who already has baseline low
levels of Protein C (from a hereditary protein C deficiency), he/she are more prone to develop
warfarin skin necrosis
 For a patient who develops warfarin-induced skin necrosis, immediate intervention is required to
prevent rapid progression of the necrosis and to minimize complications. Warfarin must be stopped
immediately. Vitamin K and therapeutic heparin anticoagulation is provided for anticoagulation.
Lastly, Protein C concentrate or FFP should be given.
43) A 62-year-old female has been on warfarin 5 mg po daily for a recently diagnosed
pulmonary embolism. She was bridged for 5 days with Enoxaparin when she was initially
started on warfarin. The PE was felt to be provoked by a right hip surgery that she had a
few days prior to her developing this clotting event, and she will be receiving a total of 3
months of anticoagulation.
She has been on treatment for the past 2 months. She recently ate 2-3 grapefruits every day.
When her INR is recently checked in the anticoagulation clinic, it is found to be 11.5. She
has no evidence of bleeding. What should be recommended?
A. Continue therapy with warfarin, but decrease dose to 2.5 mg
B. Stop warfarin and give Vitamin K replacement
C. PCC and vitamin K
D. Stop warfarin
E. Stop warfarin and give fresh frozen plasma
43) A 62-year-old female has been on warfarin 5 mg po daily for a recently diagnosed
pulmonary embolism. She was bridged for 5 days with Enoxaparin when she was initially
started on warfarin. The PE was felt to be provoked by a right hip surgery that she had a
few days prior to her developing this clotting event, and she will be receiving a total of 3
months of anticoagulation.
She has been on treatment for the past 2 months. She recently ate 2-3 grapefruits every day.
When her INR is recently checked in the anticoagulation clinic, it is found to be 11.5. She
has no evidence of bleeding. What should be recommended?
A. Continue therapy with warfarin, but decrease dose to 2.5 mg
B. Stop warfarin and give Vitamin K replacement
C. PCC and vitamin K
D. Stop warfarin
E. Stop warfarin and give fresh frozen plasma
Correct Option: B
B) Stop warfarin and give Vitamin K replacement
Grapefruit inhibits the metabolism of warfarin. Grapefruit specifically inhibits CYP450 enzymes 1A2 and
3A4

INR Action

<4.5, no bleeding U can hold or decrease the dose


of warfarin
4.5-10, no bleeding Hold warfarin U can consider vitamin K oral
for patients with high risk of
bleeding
>10, no bleeding Hold warfarin, give 2.5-5 mg
oral vitamin K

Don’t give PCC (prothrombine complex concentrate) to a non-bleeding patient.


44) The 18-year-old male with history of atrial fibrillation secondary to mitral valve stenosis is now POD #9 from
his mitral valve replacement. He is still in the hospital recovering from post-operative complications of atrial
fibrillation and pneumonia. His Heparin infusion has been stopped. He has not started anticoagulation yet, either
for his atrial fibrillation or for his new bioprosthetic mitral valve.
His HIT antibody has come back floridly positive (2.85 OD units). In addition, his serotonin release assay came
back positive. However, he has no evidence of developing a stroke or DVT/PE. In addition, his platelet count is
starting to recover and it has increased from 65,000/mm3 to 86,000/mm3. What should be done at this time?
A. Observation. Put in his chart that he has a heparin allergy
B. Initiate Coumadin now
C. Bone marrow biopsy
D. IVC filter placement
E. Start Fondaparinux or a Direct thrombin inhibitor
44) The 18-year-old male with history of atrial fibrillation secondary to mitral valve stenosis is now POD #9 from
his mitral valve replacement. He is still in the hospital recovering from post-operative complications of atrial
fibrillation and pneumonia. His Heparin infusion has been stopped. He has not started anticoagulation yet, either
for his atrial fibrillation or for his new bioprosthetic mitral valve.
His HIT antibody has come back floridly positive (2.85 OD units). In addition, his serotonin release assay came
back positive. However, he has no evidence of developing a stroke or DVT/PE. In addition, his platelet count is
starting to recover and it has increased from 65,000/mm3 to 86,000/mm3. What should be done at this time?
A. Observation. Put in his chart that he has a heparin allergy
B. Initiate Coumadin now
C. Bone marrow biopsy
D. IVC filter placement
E. Start Fondaparinux or a Direct thrombin inhibitor
Correct Option: E
E) Start Fondaparinux or a Direct thrombin inhibitor
For HIT, one should stop all sources of heparin and then provide alternative anticoagulation until the
risk of thrombosis has been eliminated, even if the patient doesn’t have thrombotic event. The risk of
thrombosis in HITT patients who are not on Anticoagulation is 50%.
There are several non-heparin anticoagulants that can be used in a patient with HIT:
--Parenteral direct thrombin inhibitors (argatroban, bivalirudin)
- Fondaparinux: Anti Xa inhibitor.
--Direct oral anticoagulants (apixaban, edoxaban, rivaroxaban, dabigatran)
> If the decision is to shift the patient to warfarin, warfarin should be started AFTER platelet
recovery (platelet >150,000.
 
Duration of Anticoagulation in HIT:
> No thrombosis: 4 weeks.
> Thrombosis 3 months.
45) A 19-year-old male with no past medical history acutely develops blood diarrhea and
presents emergently to the ER. He is admitted and an extensive workup is performed. Five
days into his hospitalization, he develops oliguria, kidney failure and thrombocytopenia. He
has no clinical signs of bleeding. A blood smear is reviewed and a few schistocytes are seen.
What therapy should be recommended?
 

A. Eculizumab
B. Plasmapheresis
C. Antibiotics
D. Supportive care
E. Platelet transfusion
45) A 19-year-old male with no past medical history acutely develops blood diarrhea and
presents emergently to the ER. He is admitted and an extensive workup is performed. Five
days into his hospitalization, he develops oliguria, kidney failure and thrombocytopenia. He
has no clinical signs of bleeding. A blood smear is reviewed and a few schistocytes are seen.
What therapy should be recommended?
 

A. Eculizumab
B. Plasmapheresis
C. Antibiotics
D. Supportive care
E. Platelet transfusion
Correct Option: D
D) Supportive care
The hemolytic uremic syndrome (HUS) is characterized by the simultaneous occurrence of
microangiopathic hemolytic anemia, thrombocytopenia and acute kidney injury. The most common
cause of HUS is Shiga toxin-producing Escherichia coli (STEC).
This patient likely has typical HUS which is characterized by the O157:H7 E. Coli infection. The
latter expresses verotoxin/Shiga-like toxin.
Initially, the bloody diarrhea is seen and this is followed 5-10 days later by HUS. The Hemoglobin is
usually < 8 g/dl and the Coombs test is negative.
Blood smear typically shows schistocytes.
There is no known therapy to prevent progression from the bloody diarrhea phase (acute infectious) to
the postdiarrheal phase.
 
 
 Treatment of E. coli–associated typical HUS is generally supportive; the use of antibiotics may
lead to increased toxin release and should be avoided. Anti-diarrhea medications should also
not be given. Patients need copious hydration and possibly pRBC transfusions depending on
their level of anemia. HUS may require dialysis during the acute phase.
A benefit for plasma exchange in typical HUS has not been demonstrated. The use of
Eculizumab in this disorder is under investigation.
 
Antithrombotic agents or oral Shiga toxin-binding agents are not recommended.
46) A 78-year-old man recently diagnosed with acute myeloid leukemia presents for cycle four
of induction therapy. A complete blood count performed showed a hemoglobin of 6.0 g/dL and a
platelet count of 101,000. The patient was feeling short of breath, weak, and dizzy. The decision
was made to transfuse one unit of packed red blood cells. Which of the following is the best
strategy to minimize the chances of graft-versus-host reaction?

a) Leukoreduction
b) Pooling
c) Plasma reduction/washing
d) Irradiation
46) A 78-year-old man recently diagnosed with acute myeloid leukemia presents for cycle four
of induction therapy. A complete blood count performed showed a hemoglobin of 6.0 g/dL and a
platelet count of 101,000. The patient was feeling short of breath, weak, and dizzy. The decision
was made to transfuse one unit of packed red blood cells. Which of the following is the best
strategy to minimize the chances of graft-versus-host reaction?

a) Leukoreduction
b) Pooling
c) Plasma reduction/washing
d) Irradiation
 Irradiation is aimed at reducing the risk of transfusion-related graft-versus-host disease. It is a disease
associated with transfusion of donor T cells which proliferate and attack the recipient.
 Gamma irradiation is used on lymphocyte-containing blood products to reduce proliferating T cells.
 Irradiation is used in patients who may be unable to mount an immune response against foreign T cells.
Such patients include leukemia and lymphoma patients, congenital immune deficiencies, hematopoietic
stem cell transplant recipients, pediatric malignancy, and aggressive chemotherapy regimens.
Prevention of transfusion-associated graft-versus-host disease is essential for blood product safety.
Transfusion-associated graft-versus-host disease is universally fatal so prevention is paramount for
patient safety. It is important to check current and institutional indications for gamma irradiation.
 Although leukoreduction minimizes leukocytes that are transfused, it is used to reduce febrile
transfusion reactions, prevent Cytomegalovirus transmission, and prevent alloimmunization to donor
HLA antigens. Transfusion-associated graft-versus-host disease (TA-GVHD) can still occur in
leukoreduced products. Gamma irradiation is used to reduce the risk of TA-GVHD.
47) A 54-year-old patient with a history of alcohol abuse presents to the emergency department
with coffee-ground emesis and melena. Complete blood count reveals hemoglobin of 5.4 g/dL,
hematocrit of 18%, and a platelet count of 32,000. Physical exam revealed an afebrile,
tachycardic, and jaundiced patient. Packed red blood cells, fresh frozen plasma, and platelets
were ordered and infused. Shortly after the infusion of the blood products, the patient became
febrile, tachycardic, and hypotensive. Vancomycin and piperacillin/ tazobactam were
administered and the patient was stabilized. The blood cultures drawn during this episode come
back positive for Staphylococcal sp. Which of the following blood products is most often
associated with the transfusion reaction seen in this patient?
a) Whole blood
b) Packed red blood cells
c) Platelets
d) Fresh frozen plasma
a) Whole blood
b) Packed red blood cells
c) Platelets
d) Fresh frozen plasma
The signs of a bacterial associated transfusion reaction
include an elevated fever, hypotension, and tachycardia.
Clinical suspicion of a possible septic transfusion reaction is
necessary for timely recognition and treatment. It is
important to know that platelets have the greatest risk in
order to raise clinical suspicion when warranted. Platelets are
associated with the greatest risk of transfusion-associated
bacterial infection due to their storage.
Platelets are stored at room temperature. Storage at room
temperature promotes bacterial growth and increases the
chance of bacterial infection during transfusion.
This increased risk and storage at room temperature is one of
the reasons that the shelf-life of platelets is only 5 days.
Several organisms can be found in contaminated platelets.
Commonly the organisms include normal skin flora
which would respond to treatment with vancomycin and
transient bacteremia with other species including gram-
negative Yersinia enterocolitica which would respond to
treatment with piperacillin/ tazobactam. Other transfusion
reactions associated with fever, tachycardia, and hypotension
would not respond to antibiotic administration.
48) A 35-year-old male presents with a history of recurrent sinopulmonary infection since
childhood. He also has a history of recurrent allergies. His family history is significant for his
elder brother with similar complaints and he eventually was diagnosed with lymphoma. The
patient is supposed to undergo an appendicectomy. The on-call surgeon wants to keep 2 units of
blood products on hold for the surgery in case if he needs them. Which of the following is the
most appropriate blood product to arrange for this patient?
 
a) Washed blood products
b) Autologous blood products
c) Gamma-irradiated blood products
d) Leukoreduced blood products
48) A 35-year-old male presents with a history of recurrent sinopulmonary infection since
childhood. He also has a history of recurrent allergies. His family history is significant for his
elder brother with similar complaints and he eventually was diagnosed with lymphoma. The
patient is supposed to undergo an appendicectomy. The on-call surgeon wants to keep 2 units of
blood products on hold for the surgery in case if he needs them. Which of the following is the
most appropriate blood product to arrange for this patient?
 
a) Washed blood products
b) Autologous blood products
c) Gamma-irradiated blood products
d) Leukoreduced blood products
 IgA is important in immune functioning of the mucosal barrier. 
 This is one of the most common immunodeficiency in human (1:100)
 Only minority of patients are symptomatic. Typical manifestation is recurrent sinopulmonary
infections, autoimmune disease and allergic diseases. 
 Anaphylactic reaction following transfusion of blood products can occur in patients with IgA
deficiency. The suitable products for patients with selective IgA deficiency are those prepared from
other IgA-deficient individuals or saline-washed red blood cells. Washing the red blood cell
reduces the immunoglobulins including IgA. This reduces the risk of allergic/anaphylactic reaction
in these patients when they are transfused blood.
49) A 57-year-old male with alcoholic liver cirrhosis presents to the emergency department after
2 days of hematemesis. He currently is not taking any medications. A previous chart shows
esophageal varices listed in the history. Today's CBC shows a hemoglobin of 5.8 g/dL. The
prothrombin time and INR are 22.5 and 1.7, respectively. One unit of packed red blood cells and
one unit of fresh frozen plasma are ordered to transfuse. Within 3 hours of transfusion, the
patient becomes dyspneic, and the lungs sound very congested. His blood pressure is 95/62
mmHg and heart rate is 112 beats/min. An emergent chest x-ray and arterial blood gas are
ordered. What is the best next step?
 
a) Intubate the patient
b) Stop the transfusion and notify the blood bank
c) Continue the transfusion and give IV furosemide 40 mg emergently
d) Slow the rate of blood transfusion
49) A 57-year-old male with alcoholic liver cirrhosis presents to the emergency department after
2 days of hematemesis. He currently is not taking any medications. A previous chart shows
esophageal varices listed in the history. Today's CBC shows a hemoglobin of 5.8 g/dL. The
prothrombin time and INR are 22.5 and 1.7, respectively. One unit of packed red blood cells and
one unit of fresh frozen plasma are ordered to transfuse. Within 3 hours of transfusion, the
patient becomes dyspneic, and the lungs sound very congested. His blood pressure is 95/62
mmHg and heart rate is 112 beats/min. An emergent chest x-ray and arterial blood gas are
ordered. What is the best next step?
 
a) Intubate the patient
b) Stop the transfusion and notify the blood bank
c) Continue the transfusion and give IV furosemide 40 mg emergently
d) Slow the rate of blood transfusion
Transfusion associated acute lung injury (TRALI)

What happens is you have an active neutrophil in the recipient (because of the underlying
clinical condition), 
The recipient receives plasma that has antibodies (donor anti-leukocytes antibodies)and
cytokines… This result in overactivation of neutrophils and release of cytokines >>> Lung
damage (TRALI)
Can occur with any blood component but the higher concentration of donor plasma, the higher
the risk. .
So FFP has the highest risk of causing TRALI. 
 The best treatment for transfusion-related acute lung injury (TRALI) is to stop the transfusion.
The blood transfusion will worsen the patient's condition. TRALI usually is associated with
plasma components such as platelets and fresh frozen plasma. Supportive care is the mainstay
of TRALI treatment. Oxygen supplementation is needed. Also, IV fluids and vasopressors are
needed for blood pressure support. Intubating the patient and protecting the airway is next step
if the patient's respiratory status worsens. Diurectics should be avoided in TRALI treatment.
50) A 56-year-old male presents to ER emergently for acute onset of melena. He is bleeding
profusely. CBC with Plt, Renal function, and DIC panel are all WNL. Over the course of 1 day,
he has received over 10 units of pRBCs. He had an urgent EGD that reveals a vascular
malformation in the stomach that had ruptured. This area was cautrized. Initially the melena did
improve. Unfortunately, two days later, he starts to bleed again in his stool, but this time he also
develops bleeding in his mouth and nose. This patient has no history of bleeding. Which of the
following labs should be checked at this time?

A) PT/PTT and fibrinogen.


B) Factor II.
C) Renal function
D) Review a blood smear
E) Bleeding time
50) A 56-year-old male presents to ER emergently for acute onset of melena. He is bleeding
profusely. CBC with Plt, Renal function, and DIC panel are all WNL. Over the course of 1 day,
he has received over 10 units of pRBCs. He had an urgent EGD that reveals a vascular
malformation in the stomach that had ruptured. This area was cautrized. Initially the melena did
improve. Unfortunately, two days later, he starts to bleed again in his stool, but this time he also
develops bleeding in his mouth and nose. This patient has no history of bleeding. Which of the
following labs should be checked at this time?

A) PT/PTT and fibrinogen.


B) Factor II.
C) Renal function
D) Review a blood smear
E) Bleeding time
Correct answer is A, A) PT, PTT, Fibrinogen
 This patient was administered many transfusions of red blood cells. These products lack platelets and
coagulation factors. His current bleeding issue could be from dilutional coagulopathy.
 Dilutional coagulopathy refers to alterations in the coagulation system that are induced by a patient
receiving large volumes of intravenous fluids or unbalanced component blood administration.
Massive transfusion occurs when there is replacement of a patient’s blood volume in <24 hours; it
takes about 10 pRBC transfused in <24 hours to cause this. It can lead to coagulopathy, hypocalcemia
and hypothermia.
This patient should have a DIC panel sent for. If dilutional coagulopathy is confirmed, then the patient
should receive FFP infusions.
 
 Infusion of FFP is indicated in the following conditions:

A) TTP/HUS ( if plasmapheresis is not available).


 B) Massive transfusions
 C) PT/PTT > 1.5 ULN with active bleeding.
 D) Reversal of warfarin overdose in bleeding patient {Though PCC (prothrombine Complex concentrate)is
preferred if available}.
 Goals of FFP administration:

 
1. Keep the fibrinogen > 100 mg/dL
2. PT/PTT should be < 1.5 the normal value
51) A 71-year-old male is currently undergoing intermittent pRBC transfusions for
myelodysplastic syndrome. His White blood count and Platelets are essentially within normal
limits. His Hb is 6.8 g/dl. He has required 2 units of pRBC every 2 months during the past two
years.
During each of the past 4 pRBC transfusions, he has had febrile non-hemolytic reactions. His
Hb is currently 7.9 g/dl and he needs another pRBC transfusion as he is very fatigued. Which
of the following may reduce his episodes of developing a febrile non-hemolytic reaction?
A. Tylenol and Benadryl prior to pRBC transfusion
B. Solumedrol 125 mg prior to the pRBC transfusion
C. Leukoreduction of RBC product
D. Gamma radiation of RBC product
E. Washing of RBC product
51)A 71-year-old male is currently undergoing intermittent pRBC transfusions for
myelodysplastic syndrome. His White blood count and Platelets are essentially within normal
limits. His Hb is 6.8 g/dl. He has required 2 units of pRBC every 2 months during the past two
years.
During each of the past 4 pRBC transfusions, he has had febrile non-hemolytic reactions. His
Hb is currently 7.9 g/dl and he needs another pRBC transfusion as he is very fatigued. Which
of the following may reduce his episodes of developing a febrile non-hemolytic reaction?
A. Tylenol and Benadryl prior to pRBC transfusion
B. Solumedrol 125 mg prior to the pRBC transfusion
C. Leukoreduction of RBC product
D. Gamma radiation of RBC product
E. Washing of RBC product
Correct Option: C
C) Leukoreduction of RBC product
Leukoreduction can take place on both red blood cell and platelet product. This removes the vast
majority of white cells in the blood product. Febrile reactions that occurs from transfusions are due to
the leukocytes that are contained in the blood product.
Passenger leukocytes are the main cause of alloimmunization to HLA and leukocyte-specific antigens
that occurs in transfusion recipients. This will cause the following:
Increase the risk for lung injury.

Platelet refractoriness.

Transmission of CMV.

Febrile nonhemolytic transfusion reaction.

 
Indications to perform leukoreduction of red blood cell product:
--Reduce HLA alloimmunization to leukocyte antigens. This includes transplants candidates and those
patients who are transfusion-dependent
 --Reduce the rate of CMV transmission as CMV is contained in white blood cells (especially in bone
marrow and solid organ transplant).
  
--Reduce febrile non-hemolytic transfusion reactions
 
Transfusion Scenario Prevention
Anaphylactic transfusion reaction Rapid onset anaphylaxis IgA deficient blood product
following transfusion in IgA should only be used
deficient patient
Urticarial (allergic) reaction Hives and allergy symptoms Diphenhydramine/
following transfusion Acetaminophen

Acute hemolytic transfusion Hemolytic reaction within 24 hrs Use ABO compatible blood
reaction of blood transfusion due to ABO
mismatch
Delayed hemolytic transfusion Delayed hemolysis (days to
reaction weeks) after transfusion, gradual,
sometime clinically silent
(detected incidentally on CBC
check). This is due to non-ABO
mismatch (Rh or Kidd blood
group)
52)A 56-year-old male is s/p day +45 allogeneic stem cell transplant for consolidation
therapy for his high-risk AML. He achieved a complete remission with chemotherapy.
Unfortunately, he recently develops melena and his hemoglobin drops from 11.2 g/dl to 7.2
g/dl. He undergoes an endoscopy and he is found to have a gastric ulcer that is causing the
bleeding. There is no endoscopic signs of GVHD. The patient is set up for 2 units of
irradiated red blood cells. What is the purpose of irradiating the red blood cells?
A. It will eradicate any CMV
B. It will prevent bacterial contamination
C. It will prevent transfusion-associated GVHD
D. It will kill any incompatible minor red blood cell antigens
E. It will destroy any incompatible plasma proteins
52) A 56-year-old male is s/p day +45 allogeneic stem cell transplant for consolidation
therapy for his high-risk AML. He achieved a complete remission with chemotherapy.
Unfortunately, he recently develops melena and his hemoglobin drops from 11.2 g/dl to 7.2
g/dl. He undergoes an endoscopy and he is found to have a gastric ulcer that is causing the
bleeding. There is no endoscopic signs of GVHD. The patient is set up for 2 units of
irradiated red blood cells. What is the purpose of irradiating the red blood cells?
A. It will eradicate any CMV
B. It will prevent bacterial contamination
C. It will prevent transfusion-associated GVHD
D. It will kill any incompatible minor red blood cell antigens
E. It will destroy any incompatible plasma proteins
Correct Option: C
C) It will prevent transfusion-associated GVHD.
What happens is an immunocompromised patient, gets blood transfusion that has donor lymphocytes.
Because he is immunodeficient, his immune system won’t be able to destroy donor lymphocytes.
Donor lymphocytes will start to attack the body and cause GVHD.  
To prevent transfusion-related GVHD, one can perform irradiation of the red blood cells or platelets.
25 Gy of gamma irradiation will block replication of any donor T lymphocytes.  
Transfusion related GVHD is a clinically devastating condition and occurs when there is engraftment
and expansion of donor T lymphocytes inside of a host. It can occur over 4 weeks after a transfusion
and patients can develop diarrhea, hepatitis, Nausea/vomiting and a rash. GVHD can target
hematopoietic cells along with other tissues.
 
 
# Indications to perform gamma irradiation of a blood product include:
  
--Individuals with congenital immunodeficiency states
 
--Individuals treated with intensive chemotherapy.
 
--Recipients of hematopoietic stem cell transplant (autologous or allogeneic)
  
--Individuals at risk for partial HLA matching with the donor due to directed donations, HLA-
matched products, or genetically homogeneous populations.
53) A23-year-old man presents with diffuse bruising. He otherwise feels well. He takes no
medications, does not use dietary supplements, and does not use illicit drugs. His medical history is
negative for any prior illnesses. He is a college student. A blood count reveals an absolute neutrophil
count of 450/μL, hemoglobin of 6 and platelet count of 21,000/μL. Bone marrow biopsy reveals
hypocellularity with a fatty marrow. Chromosome studies of peripheral blood and bone marrow cells
are performed that exclude Fanconi’s anemia and myelodysplastic syndrome. The patient has a fully
histocompatible brother. Which of the following is the best therapy?
A)Antithymocyte globulin plus cyclosporine
B) Glucocorticoids
C) Growth factors
D) Hematopoietic stem cell transplant
E) Red blood cell and platelet transfusion
53) A23-year-old man presents with diffuse bruising. He otherwise feels well. He takes no
medications, does not use dietary supplements, and does not use illicit drugs. His medical history is
negative for any prior illnesses. He is a college student. A blood count reveals an absolute neutrophil
count of 450/μL, hematocrit of 18%, and platelet count of 21,000/μL. Bone marrow biopsy reveals
hypocellularity with a fatty marrow. Chromosome studies of peripheral blood and bone marrow cells
are performed that exclude Fanconi’s anemia and myelodysplastic syndrome. The patient has a fully
histocompatible brother. Which of the following is the best therapy?
A)Antithymocyte globulin plus cyclosporine
B) Glucocorticoids
C) Growth factors
D) Hematopoietic stem cell transplant
E) Red blood cell and platelet transfusion
 Pancytopenia + empty bone marrow (filled with fat) + no obvious cause = Aplastic anemia.

 Treatment of aplastic anemia is age dependent:


 Young ( <40): Allogenic bone marrow transplant.
 Old ( >40): Immunosuppression (antithymocyte globulin+ cyclosporin + eltrombobag).
54) A 70-year-old male presents to the emergency department for evaluation of upper respiratory
symptoms with low-grade fevers. He appears clinically well. Physical examination is notable for 1 cm
palpable lymph nodes in the bilateral neck and axilla. His spleen tip is palpable just below the left
costal margin. Laboratory studies reveal a white blood cell count of 18,000/μL with 75% lymphocytes
with smudge cells, 15% neutrophils, and 5% monocytes. Chemistries are normal, including an LDH.
What is the next most appropriate step?

A) Obtain peripheral blood flow cytometry.


B) Refer for excisional lymph node biopsy.
C) Order a PET/CT to evaluate the best node for biopsy.
D) Perform a bone marrow aspiration and biopsy.
E) Treat with antibiotics and recheck a CBC in 1 month
54) A 70-year-old male presents to the emergency depart- ment for evaluation of upper respiratory
symptoms with low-grade fevers. He appears clinically well. Physical examination is notable for 1 cm
palpable lymph nodes in the bilateral neck and axilla. His spleen tip is palpable just below the left
costal margin. Laboratory studies reveal a white blood cell count of 18,000/μL with 75% lymphocytes
with smudge cells, 15% neutrophils, and 5% monocytes. Chemistries are normal, including an LDH.
What is the next most appropriate step?

A) Obtain peripheral blood flow cytometry.


B) Refer for excisional lymph node biopsy.
C) Order a PET/CT to evaluate the best node for biopsy.
D) Perform a bone marrow aspiration and biopsy.
E) Treat with antibiotics and recheck a CBC in 1 month
31. ANSWER: A. Obtain peripheral blood flow cytometry.

This patient presents with the classical findings of a patient with


CLL with diffuse lymphadenopathy and splenomegaly with a
lymphocytosis and smudge cells on peripheral smear.
The diagnosis of CLL may be established using peripheral blood
flow cytometry. Excisional lymph node biopsy and bone marrow
biopsy are not necessary given the diagnosis. PET/CT is not
necessary in the evaluation of CLL.. The patient’s symptoms are
suggestive of a viral infection and antibiotics are not indicated.
55) A 45-year-old man is evaluated by his primary care physician for complaints of early satiety and weight loss.
On physical examination, his spleen is palpable 10 cm below the left costal margin and is mildly tender to
palpation. His laboratory studies show a leukocyte count of 125,000/μL with a differential of 80% neutrophils, 9%
bands, 3% myelocytes, 3% metamyelocytes, 1% blasts, 1% lymphocytes, 1% eosinophils, and 1% basophils.
Hemoglobin is 8.4 g/dL, hematocrit is 26.8%, and platelet count is 668,000/μL. A bone marrow biopsy
demonstrates increased cellularity with an increased myeloid to erythroid ratio.
Which of the following cytogenetic abnormalities is most likely to be found in this patient?
A) Deletion of a portion of the long arm of chromosome 5, del(5q)
B) Inversion of chromosome 16, inv(16)
C) Reciprocal translocation between chromosomes 9 and
22 (Philadelphia chromosome)
D) Translocations of the long arms of chromosomes 15 and 17
E) Trisomy 12
55) A 45-year-old man is evaluated by his primary care physician for complaints of early satiety and weight loss.
On physical examination, his spleen is palpable 10 cm below the left costal margin and is mildly tender to
palpation. His laboratory studies show a leukocyte count of 125,000/μL with a differential of 80% neutrophils, 9%
bands, 3% myelocytes, 3% metamyelocytes, 1% blasts, 1% lymphocytes, 1% eosinophils, and 1% basophils.
Hemoglobin is 8.4 g/dL, hematocrit is 26.8%, and platelet count is 668,000/μL. A bone marrow biopsy
demonstrates increased cellularity with an increased myeloid to erythroid ratio.
Which of the following cytogenetic abnormalities is most likely to be found in this patient?
A) Deletion of a portion of the long arm of chromosome 5, del(5q)
B) Inversion of chromosome 16, inv(16)
C) Reciprocal translocation between chromosomes 9 and
22 (Philadelphia chromosome)
D) Translocations of the long arms of chromosomes 15 and 17
E) Trisomy 12
C) PHILADELPHIA
CHROMOSOME
CML results from direct BCR-ABL translocation>> uncontrolled proliferation and discordant
maturation
CBC shows: leukocytosis with virtually all cells of the neutrophilic series from myeloblasts to
mature neutrophils.
EXAM-ORIENTED CLUES
 Chronic myeloid leukemia (CML): chronic nonspecific symptoms: fatigue and weight loss (not
severe), massive spleen on exam, Lab shows myelocytes, metamyelocytes, blast is <20%,
thrombocytosis. Treated with Imatinib.
 Burkitt’s lymphoma: Very rapid lymph node enlargement, either in the Jaw or intrabdominal. Risk
for tumor lysis (give Rasburicase).
 Mantle cell lymphoma: It can involve any lymph node but clue is (lymphomatous intestinal
polyposis: A guy with lymphadenopathy and colonic biopsy showing lymphoma).
 Follicular lymphoma: Very slow growing lymphoma (over months).

 Acute myeloid leukemia with WBC >100k and shortness of breath : Do leukopharesis immediately.

 M3: acute promyelocytic leukemia: acute leukemia+ coagulopathy (high PT/PTT, low fibrinogen).
Treated with ATRA (all Trans Retinoic acid) and Arsenic.
56) A 56-year-old woman is diagnosed with chronic myelogenous leukemia, Philadelphia
chromosome positive. Her presenting leukocyte count was 127,000/μL, and her differential shows less
than 2% circulating blasts. Her hematocrit is 21.1% at diagnosis. She is asymptomatic except for
fatigue. She has no siblings. What is the best initial therapy for this patient?

A) Allogeneic bone marrow transplant


B) Autologous stem cell transplant
C) Imatinib mesylate
D) Interferon-α
E) Leukapheresis
56) A 56-year-old woman is diagnosed with chronic myelogenous leukemia, Philadelphia
chromosome positive. Her presenting leukocyte count was 127,000/μL, and her differential shows less
than 2% circulating blasts. Her hematocrit is 21.1% at diagnosis. She is asymptomatic except for
fatigue. She has no siblings. What is the best initial therapy for this patient?

A) Allogeneic bone marrow transplant


B) Autologous stem cell transplant
C) Imatinib mesylate
D) Interferon-α
E) Leukapheresis
 Imatinib which is a tyrosine kinase inhibitor is the treatment of choice for patients with CML.
57) A 22-year-old college student notes increasing fatigue over the past few weeks that she attributes to staying up late
studying for final examinations. Over the past 2–3 days she has noted several new bruises on her upper and lower
extremities. She does not recall antecedent trauma leading to the bruising. This morning she developed a bloody nose
lasting 20 minutes. She presents to Student Health for evaluation. Physical examination is notable for scattered quarter-
sized bruises on her upper arms and thighs. Lab studies are notable for a white blood cell count of 0.5 K/μL,
hemoglobin of 7 g/dL, and platelets of 50,000/μL. A peripheral smear is notable for 80% large atypical cells with
folded, bilobed, kidney-shaped nucleoli and elongated bluish-red rods within the cytoplasm. What translocation is the
defining feature of this disease?
A. t(11;14)
B. t(15;17)
C. t(9;22)
D. t(8;14)
E. t(14;18)
57) A 22-year-old college student notes increasing fatigue over the past few weeks that she attributes to staying up late
studying for final examinations. Over the past 2–3 days she has noted several new bruises on her upper and lower
extremities. She does not recall antecedent trauma leading to the bruising. This morning she developed a bloody nose
lasting 20 minutes. She presents to Student Health for evaluation. Physical examination is notable for scattered quarter-
sized bruises on her upper arms and thighs. Lab studies are notable for a white blood cell count of 0.5 K/μL,
hemoglobin of 7 g/dL, and platelets of 50,000/μL. A peripheral smear is notable for 80% large atypical cells with
folded, bilobed, kidney-shaped nucleoli and elongated bluish-red rods within the cytoplasm. What translocation is the
defining feature of this disease?
A. t(11;14)
B. t(15;17)
C. t(9;22)
D. t(8;14)
E. t(14;18)
ANSWER: B. t(15;17)
The patient presentation is typical for Acute promyelocytic leukemia.
You should suspect acute promyelocytic leukemia when you see a patient with leukemia who has DIC.

This disease is defined by reciprocal translocation between the long arms of chromosomes 15 and 17, with the creation of a
fusion gene, PML-RARA, which links the retinoic acid receptor alpha (RARA) gene on chromosome 17 with the
promyelocytic leukemia (PML) gene on chromosome 15.
A key component of therapy is the use of all-trans retinoic acid (ATRA), which promotes the terminal differentiation of
malignant promyelocytes to mature neutrophils.
t(11;14)—Overexpression of cyclin D1 in mantle cell lymphomas is strongly associated with the t(11;14.
t(9;22)—The vast majority of patients (90%–95%) demonstrate the t(9;22)(q34;q11.2) reciprocal translocation that results in
the Ph chromosome-BCR-ABL1 fusion gene or its product, the BCR-ABL1 fusion mRNA. The BCR-ABL1 fusion gene is the
target of therapies such as dasatanib, nilotinib, and imatinib.
t(8;14)—Burkitts Lymphoma is associated with a translocation between the long arm of chromosome 8, the site of the c-MYC
oncogene (8q24), and one of three locations on immunoglobulin (Ig) genes. The most common translocation is with the the Ig
heavy chain gene on chromosome 14 (approximately 80%).
t(14;18)—Approximately 85% of patients with follicular lymphoma have t(14;18), which results in the overexpression of B cell
leukemia/lymphoma 2 (BCL2), an oncogene that blocks programmed cell death (apoptosis), leading to prolonged cell survival.
58) A 60-year-old woman with past medical history of hypertension and COPD is brought to the emergency room
by her daughter. She is lethargic and is having difficulty walking. She has noted increasing difficulty breathing over
the past few hours. Physical examination is notable for temperature of 38, blood pressure of 150/70 mm Hg,
respiratory rate of 24 breaths per minute, and O2sat 88% on room air. She has difficulty with finger-to-nose and
rapid alternating movements. She has fine crackles bilaterally. A complete blood count is notable for a white blood
cell count of 110 × 106, hemoglobin of 8, and platelets of 75,000/μL. A chest X-ray shows bilateral infiltrates.
What would be the appropriate next step in management of this patient?
A) Give a unit of PRBCs.
B) Start antibiotics for possible pneumonia and follow for symptom improvement before initiating more invasive
procedures.
C) Start steroids and antibiotics for COPD flare.
D) Start leukapheresis while establishing diagnosis.
E) Establish diagnosis and start appropriate therapy.
58) A 60-year-old woman with past medical history of hypertension and COPD is brought to the emergency room
by her daughter. She is lethargic and is having difficulty walking. She has noted increasing difficulty breathing over
the past few hours. Physical examination is notable for temperature of 38, blood pressure of 150/70 mm Hg,
respiratory rate of 24 breaths per minute, and O2sat 88% on room air. She has difficulty with finger-to-nose and
rapid alternating movements. She has fine crackles bilaterally. A complete blood count is notable for a white blood
cell count of 110 × 106, hemoglobin of 8, and platelets of 75,000/μL. A chest X-ray shows bilateral infiltrates.
What would be the appropriate next step in management of this patient?
A) Give a unit of PRBCs.
B) Start antibiotics for possible pneumonia and follow for symptom improvement before initiating more invasive
procedures.
C) Start steroids and antibiotics for COPD flare.
D) Start leukapheresis while establishing diagnosis.
E) Establish diagnosis and start appropriate therapy.
ANSWER: D. Start leukapheresis while establishing diagnosis.
This patient has a WBC >100,000 with anemia and thrombocytopenia. She has a likely diagnosis of
leukemia with an elevated WBC count leading to hyperleukocytosis and leukostasis.
Leukostasis results from increased blood viscosity leading to thrombi in microcirculation. Patients are
usually symptomatic when the WBC reaches levels of >100,000. If left untreated, the 1-week mortality
rate is approximately 20% to 40%.
In the setting of her WBC and CXR findings, it is likely that she has pulmonary evidence of leukostasis,
presenting with dyspnea, tachypnea, and hypoxemia. Respiratory acidosis and cor pulmonale may also
occur. CXR can show diffuse bilateral infiltrates. Approximately 80% of patients with leukostasis are
febrile, usually as a result of associated inflammation. While starting empiric antibiotics is reasonable,
waiting to see if the patient improves would waste valuable time during which the patient can be
leukopheresed.
Transfusions would increase blood viscosity and increase the risk of worsening the respiratory
compromise and intracranial hemorrhage.
Leukapheresis should be initiated while establishing a diagnosis in patients who have symptomatic
leukostasis. It is a temporizing measure; thus, a diagnosis needs to be promptly made and definitive
therapy should be started.
59)A 30-year-old white female who is diagnosed with acute promyelocytic leukemia was started on all-
trans retinoic acid (ATRA). On the third day of the therapy, the patient developed fever, peripheral edema,
and shortness of breath. Her vital signs include a temperature of 102.3 F, blood pressure 90/58 mmHg,
pulse 112 beats/min, pulse oximetry 84% on 10 L of high flow oxygen, and respiratory rate 24
breaths/min. Pertinent laboratory studies include WBC 31,000 cells/mL, hemoglobin 7 grams/dL, platelet
count 50,000/mL, serum creatinine 2.4 mg/dL (baseline 0.9 mg/dL). A chest x-ray suggests pulmonary
edema. She was transferred to the intensive care unit for further management. Which of the following is
correct regarding the management of this condition?
 
a) Add arsenic trioxide to ATRA
b) Supportive therapy and continue ATRA
c) Continue ATRA and start hydroxyurea
d) Discontinue ATRA and start the patient on IV dexamethasone 10 mg twice daily
59)A 30-year-old white female who is diagnosed with acute promyelocytic leukemia was started on all-
trans retinoic acid (ATRA). On the third day of the therapy, the patient developed fever, peripheral edema,
and shortness of breath. Her vital signs include a temperature of 102.3 F, blood pressure 90/58 mmHg,
pulse 112 beats/min, pulse oximetry 84% on 10 L of high flow oxygen, and respiratory rate 24
breaths/min. Pertinent laboratory studies include WBC 31,000 cells/mL, hemoglobin 7 grams/dL, platelet
count 50,000/mL, serum creatinine 2.4 mg/dL (baseline 0.9 mg/dL). A chest x-ray suggests pulmonary
edema. She was transferred to the intensive care unit for further management. Which of the following is
correct regarding the management of this condition?
 
a) Add arsenic trioxide to ATRA
b) Supportive therapy and continue ATRA
c) Continue ATRA and start hydroxyurea
d) Discontinue ATRA and start the patient on IV dexamethasone 10 mg twice daily
 Differentiation syndrome is a cytokine release
syndrome seen after initiation of differentiating
agents characterized by fever, peripheral edema,
pulmonary edema, and multi-organ dysfunction.
It can be fatal, if not promptly recognized and
treated. High dose systemic corticosteroids are
helpful in these cases. All-trans retinoic acid
(ATRA) can be continued in mild cases in
addition to IV corticosteroids. ATRA must be
discontinued in severe cases with multi-organ
dysfunction and can be restarted when the
differentiation syndrome resolves.
60) A 67-year-old female is brought to the clinic with symptomatic anemia and has required
several transfusions over the past 2-3 months. Currently, her hemoglobin is 7.5 g/dL, absolute
neutrophil count 1500/microL and platelet count 119,000/microL. She had a bone marrow
biopsy performed that reveals myelodysplastic syndrome with 6% blasts and cytogenetics
positive for 5q deletion. She did not have any mutations or other cytogenetics. Which of the
following is the best initial course of treatment?
 
a) An erythropoietin stimulating agent
b) Lenalidomide
c) Hypomethylating agent
d) Stem cell transplant
 MDS is diagnosed when : Cytopenia + >10% dysplastic
cells (abnormal looking cells) + Blast <20%. 

 Remember: Blasts >20% is acute myeloid leukemia

 This is a patient requiring frequent transfusions with a


diagnosis of MDS and 5q deletion, which is generally a
favorable type of MDS. A patient with symptomatic
anemia and 5q deletion is generally a good candidate for
lenalidomide.

 Stem cell transplant may be a potential treatment down


the line, but the initial treatment of lenalidomide is
generally recommended.
 A hypomethylating agent may be used, but initial
treatment with lenalidomide in the setting of the patient's
cytogenetics should be tried first.
61) A 74-year-old woman is anemic with hemoglobin of 10.5 g/dL. She has symptoms of
tingling in her feet and back discomfort. X-rays of her spine reveal osteopenia and multiple lytic
lesions. She has an immunoglobulin G (IgG) spike on serum protein electrophoresis. Which of
the following is consistent with the patient presentation?
(A) osteoblastic bone lesions
(B) increased spleen size
(C) increased osmotic fragility
(D) increased plasma cells in bone marrow
61) A 74-year-old woman is anemic with hemoglobin of 10.5 g/dL. She has symptoms of
tingling in her feet and back discomfort. X-rays of her spine reveal osteopenia and multiple lytic
lesions. She has an immunoglobulin G (IgG) spike on serum protein electrophoresis. Which of
the following is consistent with the patient presentation?
(A) osteoblastic bone lesions
(B) increased spleen size
(C) increased osmotic fragility
(D) increased plasma cells in bone marrow
 Increased plasma cells in bone marrow.

 Remember, in MM there is an increase in osteolytic activity NOT osteoblastic bone lesions.


DIAGNOSIS OF MULTIPLE
MYELOMA
 Bone marrow >10% clonal plasma cells + one of CRAB
 C: Calcium
 R: Renal insufficiency
 A: anemia
 B: bone lytic lesion.

 > 60% clonal plasma cells in bone marrow biopsy alone qualify for diagnosis of MM.
62) A 54-year-old male is seen in the hematology clinic with symptoms of confusion, loss of
coordination, headaches, fatigue, and weight loss. He denies any illness except for diet-
controlled diabetes mellitus. He does not smoke or drink. Examination reveals several swollen
cervical lymph nodes and blood work reveals an elevated erythrocyte sedimentation rate.
Serology reveals that he has an excess level of IgM. What is the most likely diagnosis?
 
a) Multiple myeloma
b) Waldenstrom macroglobulinemia
c) Leukemia
d) Hodgkin lymphoma
62) A 54-year-old male is seen in the hematology clinic with symptoms of confusion, loss of
coordination, headaches, fatigue, and weight loss. He denies any illness except for diet-
controlled diabetes mellitus. He does not smoke or drink. Examination reveals several swollen
cervical lymph nodes and blood work reveals an elevated erythrocyte sedimentation rate.
Serology reveals that he has an excess level of IgM. What is the most likely diagnosis?
 
a) Multiple myeloma
b) Waldenstrom macroglobulinemia
c) Leukemia
d) Hodgkin lymphoma
Viscosity syndrome (shortness of breath, blurry vision, ataxia, tortuous retinal veins) + IgM +/-
lymph node enlargement : Think of Waldenstrom macroglulinemia. 

There is proliferation of IgM-producing lymphoid tissue in Waldenstrom macroglobulinemia


(WM).
LYMPHOM
ALL CLL MM
AS
naïve

B-lymphocytes
Plasma
Lymphoid cells
progenitor T-lymphocytes

AML Myeloproliferative disorders


Hematopoietic Myeloid Neutrophils
stem cell progenitor

Eosinophils

Basophils

Monocytes

Platelets

Red cells
 For exam purposes, malignancies in plasma cells are either multiple myeloma or Waldenstrom
Macroglobulinemia.

 WM : Monoclonal IgM.
 MM: Monoclonal Ig (other than IgM) : IgG, IgE, IgA.

Symptoms are different as well WM is hyperviscosity whereas MM is CRAB symptoms.

You can have IgM multiple myeloma but is extremely rare and you won’t be ask about it in the
exam.
63) A 21-year-old male presents to the clinic for a regular follow up. He was treated for acute
myeloid leukemia three years ago with chemotherapy, and complete clinical remission was
achieved after three courses. Later, however, the patient relapsed within a year, and although he
survived, the second remission was challenging to achieve. Which of the following is the best
next step in the management of this patient?
 
a) Gene therapy followed by treatment with anti-CD45 monoclonal antibody
b) Supporting immunotherapy with granulocyte colony-stimulating factor and platelet
transfusion
c) Allogenic stem cell transplant.
D) Treatment with gemtuzumab ozogamicin (Mylotarg)
63) A 21-year-old male presents to the clinic for a regular follow up. He was treated for acute
myeloid leukemia three years ago with chemotherapy, and complete clinical remission was
achieved after three courses. Later, however, the patient relapsed within a year, and although he
survived, the second remission was challenging to achieve. Which of the following is the best
next step in the management of this patient?
 
a) Gene therapy followed by treatment with anti-CD45 monoclonal antibody
b) Supporting immunotherapy with granulocyte colony-stimulating factor and platelet
transfusion
c) Allogenic stem cell transplant.
D) Treatment with gemtuzumab ozogamicin (Mylotarg)
 For board purposes, any question with relapsed AML, treatment is stem cell transplant
Please keep in mind, that this diagram is very simplified many details were left out
intentionally.
64) A 70-year-old female diagnosed with Epstein Barr virus-associated plasmablastic lymphoma
is started on chemotherapy with da-R-EPOCH (dose adjusted rituximab, etoposide, prednisone,
vincristine, doxorubicin) and has completed four cycles of chemotherapy so far. She reports
numbness and tingling in both her fingers and toes. Which of the following chemotherapy drugs
is responsible for her neuropathy?
 
a) Prednisone
b) Vincristine
c) Etoposide
d) Doxorubicin
64) A 70-year-old female diagnosed with Epstein Barr virus-associated plasmablastic lymphoma
is started on chemotherapy with da-R-EPOCH (dose adjusted rituximab, etoposide, prednisone,
vincristine, doxorubicin) and has completed four cycles of chemotherapy so far. She reports
numbness and tingling in both her fingers and toes. Which of the following chemotherapy drugs
is responsible for her neuropathy?
 
a) Prednisone
b) Vincristine
c) Etoposide
d) Doxorubicin
Neurotoxicity is a very well-known complication of vincristine which is a microtubule inhibitor.
Dose reduction should be considered in patients who are experiencing moderate symptoms of
paresthesias, sensory and motor neuropathy and limiting instrumental activities of daily living.
Duloxetine, an antidepressant can be used to treat neuropathic pain arising due to chemotherapy
such as vincristine.
Chemotherapy Side effects note

Cyclophosphamide Hemorrhagic cystitis Use Mesna

Cisplatin Nephrotoxicity, deafness can use Amifostine(not


commonly used)
Procarabazine Leukemia

Doxorubicin Cardiomyopathy Dexrazoxane is an iron chelating


agent to prevent production fo
free radicals
MTX Nephrotoxicity (crystals) Leucovorin

5-FU/Capacitabine Hand-foot syndrome

Bleomycin Pneumonitis, pulmonary fibrosis

Vincristine/Vinblastin Neurotoxicity

All Trans-retinoic acid Differentiation syndrome (give steroid for differentiation


syndrome)
used in AML-M3
Ifosfamide Neurotoxicity Give Methylene blue
65) A 22-year-old male with history of B cell acute lymphoblastic leukemia. The patient was
treated with chemotherapy a year ago but now has evidence of relapse of his disease. The patient
is being considered for chimeric antigen receptor-engineered T cells (CAR-T cells). chimeric
antigen receptor-engineered T cells (CAR-T cells) have targeted which receptor in the treatment
of acute lymphoblastic leukemia?
 
a) CD5
b) CD19
c) CD45
d) CD99
65) A 22-year-old male with history of B cell acute lymphoblastic leukemia. The patient was
treated with chemotherapy a year ago but now has evidence of relapse of his disease. The patient
is being considered for chimeric antigen receptor-engineered T cells (CAR-T cells). chimeric
antigen receptor-engineered T cells (CAR-T cells) have targeted which receptor in the treatment
of acute lymphoblastic leukemia?
 
a) CD5
b) CD19
c) CD45
d) CD99
CAR T cell therapy is basically taking the
patient own lymphocytes, reprogram it to
attack the tumor cells.
In hematology, CAR T cells are being
used in Relapsed Non-Hodgkin
lymphoma and in Relapsed acute
lymphocytic leukemia.
The target antigen is CD19.
66) A 64-year-old male was recently started on the RICE regimen [ritiximab, ifosfamide,
carboplatin, and etoposide] as salvage therapy for refractory non-Hodgkins lymphoma. About 5
hours after receiving his first dose of ifosfamide, the patient became acutely confused and
started hallucinating. Vitals signs show a blood pressure of 100/70 mmHg but physical exam is
otherwise unremarkable. Which of the following is the next best step in the management of this
patient?
 
a) Discontinuation of the offending drug only
b) Methylene blue administration
c) Plasmapharesis.
d) Observation
66) A 64-year-old male was recently started on the RICE regimen [ritiximab, ifosfamide,
carboplatin, and etoposide] as salvage therapy for refractory non-Hodgkins lymphoma. About 5
hours after receiving his first dose of ifosfamide, the patient became acutely confused and
started hallucinating. Vitals signs show a blood pressure of 100/70 mmHg but physical exam is
otherwise unremarkable. Which of the following is the next best step in the management of this
patient?
 
a) Discontinuation of the offending drug only
b) Methylene blue administration
c) Plasmapharesis.
d) Observation
Methylene blue administration has shown improved outcomes in patients who developed
metabolic encephalopathy secondary to ifosfamide. Encephalopathy is usually reversible within
72 hours. Methylene blue can be used as prophylaxis as well for patients who are at risk for
encephalopathy.
67) A 65-year-old female with congestive heart failure and chronic renal failure presents with
shortness of breath and generalized body swelling. She reports her symptoms have worsened since
yesterday. On physical exam, there is distention of the jugular venous pulse (JVP), presence of S3 on
cardiac auscultation, and the presence of crackles bilaterally on lung auscultation. There is pitting
edema of both lower legs. There are waxy papules in the axilla and inguinal region. Laboratory
analysis reveals serum creatinine of 6.1 mg/dL, total protein level of 9.2 g/dL, and blood urea nitrogen
(BUN) level of 85 mg/dL. Urine analysis reveals 3+ protein but with the absence of cells or cellular
casts. An echocardiogram shows a thickened left ventricle with preserved systolic function. Which of
the following is the most appropriate next step in the management of this patient?
 
a) Fat pad biopsy
b) Renal ultrasound
c) Right heart catheterization
d) Whole body PET scan.
67) A 65-year-old female with congestive heart failure and chronic renal failure presents with
shortness of breath and generalized body swelling. She reports her symptoms have worsened since
yesterday. On physical exam, there is distention of the jugular venous pulse (JVP), presence of S3 on
cardiac auscultation, and the presence of crackles bilaterally on lung auscultation. There is pitting
edema of both lower legs. There are waxy papules in the axilla and inguinal region. Laboratory
analysis reveals serum creatinine of 6.1 mg/dL, total protein level of 9.2 g/dL, and blood urea nitrogen
(BUN) level of 85 mg/dL. Urine analysis reveals 3+ protein but with the absence of cells or cellular
casts. An echocardiogram shows a thickened left ventricle with preserved systolic function. Which of
the following is the most appropriate next step in the management of this patient?
 
a) Fat pad biopsy
b) Renal ultrasound
c) Right heart catheterization
d) Whole body PET scan.
The waxy papules and cardiac findings make the diagnosis likely to be amyloidosis. The
subcutaneous abdominal fat aspirate is an easy and innocuous procedure that will stain Congo
red positive with apple-green birefringence and has an 81% diagnostic sensitivity in AL
amyloidosis. Fat pad biopsy is 60 to 80 percent sensitive for amyloidosis. The clinical features
of amyloidosis vary depending on which type of amyloid fibrils are responsible. Systemic
amyloidosis can lead to heart failure with left ventricular hypertrophy on echocardiogram with
standard or low voltage electrocardiogram.
68) A 55-year-old man with a history of stable chronic myelogenous leukemia (CML) is
admitted to the hospital with suspected neutropenic fever. His vital signs show blood pressure
96/64 mmHg, pulse 98/min, and temperature 39 C. Urinalysis demonstrates >30 white blood
cells and 3+ bacteria. He is given appropriate antibiotics and fluid resuscitation. The patient's
blood pressure normalizes with fluid and antibiotics, and his fever abates. The following day, the
patient is noted on morning rounds to be obtunded and jaundiced. Repeat lab work reveals
newly developed transaminitis, worsening pancytopenia, and a rising creatinine. A serum ferritin
level is 10,000 mcg/L. Which of the following bone marrow biopsy findings is most likely to
confirm the diagnosis in this patient?
a) Blast count >20%
b) Myelofibrosis
c) Hemophagocytosis
d) Ringed sideroblasts
68) A 55-year-old man with a history of stable chronic myelogenous leukemia (CML) is
admitted to the hospital with suspected neutropenic fever. His vital signs show blood pressure
96/64 mmHg, pulse 98/min, and temperature 39 C. Urinalysis demonstrates >30 white blood
cells and 3+ bacteria. He is given appropriate antibiotics and fluid resuscitation. The patient's
blood pressure normalizes with fluid and antibiotics, and his fever abates. The following day, the
patient is noted on morning rounds to be obtunded and jaundiced. Repeat lab work reveals
newly developed transaminitis, worsening pancytopenia, and a rising creatinine. A serum ferritin
level is 10,000 mcg/L. Which of the following bone marrow biopsy findings is most likely to
confirm the diagnosis in this patient?
a) Blast count >20%
b) Myelofibrosis
c) Hemophagocytosis
d) Ringed sideroblasts
Hemophagocytic lymphohistiocytosis (HLH) is a hyper-inflammatory disorder characterized by
macrophages activation and destruction of bone marrow. Although originally described and
elucidated in children, HLH can also occur in adults with some estimates as high as 1:2000 adult
admissions to the ICU. The most common triggers for developing HLH in adults is an infection,
followed closely by hematologic malignancies. This patient with a history of CML and presents
with a complicated UTI. Despite appropriate upfront treatment, they go on to develop acute
multiorgan failure that cannot be easily explained by either a resistant UTI or worsening of his
CML. Extremely elevated ferritin levels can be seen in patients who develop HLH. Serum levels
>500 mcg/L are part of the HLH-2004 diagnostic criteria; however, levels >10,000 mcg/L have
been shown to be 90% sensitive and 98% specific in a Texas Children's Hosptial cohort of
pediatric cases.
Thank you.

For any further questions please email me on


Fad.alshaibani@gmail.com

Best of luck

You might also like